You are on page 1of 115

AIPG-2016

( exam conducted in Dec 2015)

DADH
1..Alternative light and dark bands seen in tooth cross section are known as?

a.Hunter suchrenger band

b.Incremental lines of retzius

c.Neonatal line

d.Reversal line

Ans: a.Hunter suchrenger band

Refer: Essentials of Oral Histology and Embryology: A Clinical Approach, 4th


Edition,page no.100

Hunter-Schreger bands: Changes in the direction of enamel rods that minimize cleavage in
the axial direction produce an optical appearance called HunterSchreger bands. They have
alternate light and dark zones, which have slightly different permeability and organic content.

2.Development of supernumerary tooth occurs in which stage?

a.Apposition

b.Initiation

c.Calcification

d.Dental lamina

Ans: b.Initiation

Refer: Principles and Practice Of Pedodontics - Page 84, Arathi Rao,3rd e


Abnormalities in physiologic stages of tooth development
Tooth development is divided into 4 stages based on physiological process of tooth growth:

Initiation: It is a part of Bud Stage. For Initiation Epithelial Ectomesenchymal interactions


are essential. Initiation is the first step in the development of tooth. Each tooth has its own
initiation time and it is not know how this time is determined. Even in case of teeth located in
abnormal positions initiation is started on time.
Proliferation: As the name says Proliferation in tooth development is increase in Size and
Shape of the developing tooth germ and the cells. Which have the ability to differentiate into
various structures based on stimulation in the later stages of tooth development.
Histodifferentiation: Seen in the early Bell Stage of tooth developement. The formative
cells of the tooth germ undergoes many functional and morphologic changes to transform
into functional cells. Which gives the cells an ability to multiply. the peripheral cells of the
dental papilla differentiate into odontoblasts which produces dentin. And as we know that
presence of Dentin stimulates the presence of Enamel. In the absence of Dentin, Enamel
formation does not occur.
Morphodifferentiation: The dentinoenamel junction and the cementoenamel junction are
developed by the continuous deposition of enamel, dentin and cementum from ameloblasts,
odontoblasts and cementoblasts. Which results in the complete formation of Morphologic
outline, pattern, size and shape of the tooth.
Apposition: Apposition is the addition of new layers on teh existing ones and after the
morphologic part of tooth development is completed, a rhythmic, layer like deposition of an
extra cellular matrix of enamel and dentin takes place. It is charecterized by a regular,
rhythmic deposition of extracellular matrix. In this stage periods of Activity and Periods of
Rest are seen.

Supernumerary teeth in the deciduous dentition are usually normal or conical shaped,
whereas supernumerary teeth in the permanent dentition can exhibit various shapes. They
may have normal morphology or may be rudimentary and miniature with little or no
resemblance to the other teeth . Based on their morphology, supernumerary teeth are
classified into four types, including conical type, tuberculate type, supplemental teeth, and
odontomas . The most common supernumerary teeth are small conical peg-shaped with root
development at the similar stage or ahead of that of adjacent teeth. They usually develop in
the anterior maxilla as mesiodens. Tuberculate supernumerary teeth are large barrel-shaped
with multiple cusps or tubercles. Their root development is delayed compared to that of
adjacent teeth. They are mostly found unerupted in the palatal aspect of the maxillary central
incisors, and this can cause the impaction of permanent maxillary incisors.

3.Histologically Pulp tissue is?

a.Nervous Tissue

b.Blood Tissue

c.Loose Connective tissue

d.Granulomatous tissue

Ans: c.Loose Connective tissue

Refer: Orban's Oral Histology & Embryology - Page 2,13th e

Dental pulp is a loose connective tissue with an appearance similar to mucoid CT. It contains
the components common to all connective tissues:
Cells: fibroblasts and undifferentiated mesenchymal cells as well as other cell types
(macrophages, lymphocytes, etc.) required for the maintenance and defense of the
tissue .
Fibrous matrix: collagen fibers, type I and II, are present in an unbundled and
randomly dispersed fashion, higher in density around blood vessels and nerves. Type
I collagen is thought to be produced by the odontoblasts as dentin, secreted by these
cells, is composed of type I collagen. Type II is probably produced by the pulp
fibroblasts as this type increases in frequency with the age of the tooth. Older pulp
contains more collagen of both the bundled and diffuse types.

Ground substance: the environment that surrounds both cells and fibers of the pulp
is rich in proteoglycans, glycoproteins and large amounts of water.

4.Mechanism of invasion of bacteria into dentinal tubules?

a.By lowering PH

b.By changing the hydrodynamic flow within dentinal tubules.

c.Increase mobility of micro-organism in dentinal tubules.

d.By utilizing calcium for fertilization.

Ans: c.Increase mobility of micro-organism in dentinal tubules.

Refer: Cohen's Pathways of the Pulp Expert Consult - Page 602,11th e

Bacterial invasion of dentinal tubules commonly occurs when dentin is exposed following a
breach in the integrity of the overlying enamel or cementum. Bacterial products diffuse
through the dentinal tubule toward the pulp and evoke inflammatory changes in the pulpo-
dentin complex. These may eliminate the bacterial insult and block the route of infection.
Unchecked, invasion results in pulpitis and pulp necrosis, infection of the root canal system,
and periapical disease. While several hundred bacterial species are known to inhabit the oral
cavity, a relatively small and select group of bacteria is involved in the invasion of dentinal
tubules and subsequent infection of the root canal space. Gram-positive organisms
dominate the tubule microflora in both carious and non-carious dentin. The relatively high
numbers of obligate anaerobes present-such as Eubacterium spp., Propionibacterium spp.,
Bifidobacterium spp., Peptostreptococcus micros, and Veillonella spp.-suggest that the
environment favors growth of these bacteria. Gram-negative obligate anaerobic rods, e.g.,
Porphyromonas spp., are less frequently recovered. Streptococci are among the most
commonly identified bacteria that invade dentin.

PHARMACOLOGY

5.A patient has nephropathy ,which of the following drug is avoided in this patient?
a.Acetaminophen

b.Acetyl silicic acid

c.NSAIDS

d.Opioids

Ans: c.NSAIDS

Refer: Adverse Drug Reactions - Page 471, By Anne Lee,2nd e

Nephrotoxic drugs

Drugs causing prerenal damage


Drugs that cause excessive gastrointestinal losses, either through diarrhoea or
vomiting, also cause volume depletion and may precipitate acute kidney injury.
NSAIDs, even in short courses, can cause acute kidney injury as a result of renal
underperfusion.

Angiotensin-converting enzyme (ACE) inhibitors can also cause a deterioration in


renal function. However, this is a problem only in patients with compromised renal
perfusion, particularly those with renal artery stenosis.

Care should be taken when an ACE inhibitor and NSAID are prescribed together, as
this combination may precipitate an acute deterioration in renal function.

Drugs causing intrarenal damage


Intrarenal damage may result in a direct toxic effect on the kidneys or hypersensitivity
reactions.
Most drugs that cause damage within the kidneys do so as a result of hypersensitivity
reactions, which involve either glomerular or interstitial damage.

Drugs that have been reported to cause glomerulonephritis include penicillamine,


gold, captopril, phenytoin and some antibiotics, including penicillins, sulphonamides
and rifampicin.

Drugs that may cause interstitial nephritis include penicillins, cephalosporins,


sulfonamides, thiazide diuretics, furosemide, NSAIDs and rifampicin.

There are a number of drugs that cause direct toxicity to the renal tubules (acute
tubular necrosis) - eg, aminoglycosides, amphotericin and ciclosporin.

Drugs causing postrenal damage (urinary tract obstruction)


High-dose sulfonamides, acetazolamide or methotrexate may cause crystalluria and
could therefore cause urinary tract obstruction.
Anticholinergics (eg, tricyclic antidepressants), and alcohol may cause urinary tract
obstruction due to retention of urine in the bladder.

Other nephrotoxic drugs


Cephalosporins: cephaloridine, one of the first cephalosporins introduced, has been
associated with direct renal toxicity and is no longer in clinical use. Other
cephalosporins are much less likely to produce renal damage but third-generation
cephalosporins (eg, cefixime) have very rarely been reported to cause nephrotoxicity.
Analgesics:

o NSAIDs may cause acute kidney injury due to hypoperfusion and interstitial
nephritis, as well as analgesic nephropathy (chronic interstitial nephritis and
papillary necrosis).

o Analgesic nephropathy has been most commonly seen with combination


analgesic products that contain aspirin and/or paracetamol.

o Analgesic nephropathy is one of the few preventable causes of chronic kidney


disease. Discontinuation of the drugs often results in stabilisation or even
improvement in renal function but continued use leads to further renal
damage.

Lithium: serum levels of lithium consistently above the therapeutic range have been
associated with development of a nephrogenic diabetes insipidus.

6.Best Intravenous agent for recovery in day care anesthesia is?

a.Thiopentone

b.Sevoflurane

c.Isoflurane

d.Propofol

Ans:d

Refer: Textbook of Oral and Maxillofacial Surgery - Page 202, Neelima Anil Malik,3rd e

Best Intravenous agent for recovery in day care anesthesia is Propofol. Although propofol is
the gold standard drug in day-care procedures, it has its own side effects like apnea,
cardiovascular instability, pain on injection, as well as its cost. The ideal drug combination to
achieve this end remains elusive. Therefore, a combination of propofol, thiopentone, and
ketamine may be a better alternative.

7.A 4 year old child with restricted mouth opening inhalation anesthetic of choice?

a.Isoflurane

b.Sevoflurane

c.Desflurane

d.Halothane

Ans: c.Desflurane
Refer: Clinical Anesthesia - Page 438, Paul G. Barash,6th e

In small children, gaseous induction using sevoflurane (with parental presence) is often
easiest. Since its introduction, sevoflurane has largely replaced halothane as agent of choice
because inhalation is quick and smooth and there are limited cardiovascular and respiratory
effects.Sevoflurane supplementation of 66% nitrous oxide in oxygen is used. Sevoflurane
may either be introduced in 2% increments every 2 to 3 breaths to a maximum of 8%, with
maintenance of anaesthesia at or around 4%, or it may be introduced at the maximum
concentration of 8%, with maintenance at 4%. Induction using 8% sevoflurane does not
appear to cause any adverse effects.However, if sevoflurane is not available halothane is
preferred over isoflurane that is irritant and can lead on to coughing and laryngospasm.

8.Best inhalational anesthesia for day care surgery?

a.Halothane

b.Isoflurane

c.Sevoflurane

d.Desflurane

Ans: d.Desflurane

Refer: Textbook of Oral and Maxillofacial Surgery - Page 177, Rajiv M Borle,1st e

Best inhalational anesthesia for day care surgery Desflurane.Both sevoflurane and
isoflurane are suitable for day care anaesthesia. Sevoflurane has advantages of quicker
emergence, early discharge from PACU I and less airway hypersensitivity. Isoflurane has
more incidence of airway hyper reactivity though milder level, when compared to
sevoflurane. Incidence of nausea and vomiting and requirement of postoperative analgesia
are comparable in both the groups. Isoflurane is less costly in comparison to sevoflurane.

9.Which of the following inducing anesthetic agent also have antiemetic property?

a.Propofol

b.Thiopentone

c.Etomidate

d.Fentanyl

Ans: a.Propofol

Refer: Goodman & Gillman ,12th e /503/Peterson's Principles of Oral and Maxillofacial
Surgery,3rd e ,page no 60.
Propofol is the most widely used i.v. anesthetic agent for induction. It is highly lipophilic and
distributes rapidly into the CNS and other tissues, which accounts for its rapid onset of
action.Propofol produces unconsciousness within the time it takes for the drug to travel from
the injection site to the brain, which is referred to as one "arm-brain circulation time" and
requires less than one minute. The onset of anesthesia is smooth, although the drug can
cause pain at the injection site. Propofol is rapidly and extensively metabolized in the liver
and at extrahepatic sites, which means it has a high rate of total body clearance. The drug
has a direct antiemetic effect through an unknown mechanism.It also is euphorigenic, but
does not have residual psychotic effects as does ketamine.

PEDODONTICS

10..According to latest theory duration of, enamel etching in primary teeth when compared to
permanent teeth is ?

a. Same as permanent teeth (15 sec)

b. More than permanent teeth (35 sec)

c. Less For permanent teeth (10 sec)

d. No need For etching in primary teeth

Ans : a. Same as permanent teeth (15 sec)

Refer: Textbook of Operative Dentistry - Page 246, Lloyd Baum, Ralph W. Phillips,
Melvin R. Lund,3rd e

According to latest theory duration of, enamel etching in primary teeth when compared to
permanent teeth is, same as permanent teeth (15 sec).20 sec etching time is
recommended .

11.Most acceptable property of pit and fissure sealant while selecting it is?

a.Adequate working time

b.Moderate water solubility

c.Low volatility

d.Thermal conductivity more than tooth.

Ans: a.Adequate working time

Refer: Shafer'S Textbook Of Oral Pathology (6Th Edition) - Page 464

Most acceptable property of pit and fissure sealant while selecting is Adequate working time.
The properties required of an ideal fissure sealant include biocompatibility, anticariogenicity,
adequate bond strength, good marginal integrity, resistance to abrasion and wear, and cost
effectiveness. The clinical efficiency of fissure sealants is directly related to their
retention.Retention depends on morphology of pits and fissures, adequate isolation,
conditioning of enamel, application techniques, particular material characteristics like
viscosity, surface tension, and adequate adhesion.

12.Gingival Abscess is usually common in primary teeth because?

a.Flaring of teeth towards buccal surface

b.Thin layer of dentin

c.More accessory canals are present on buccal surface with porous floor & thin bony
surface.

d.Canals in primary teeth are ribbon shaped.

Ans: c.More accessory canals are present on buccal surface with porous floor & thin bony
surface.

Refer: Textbook of Pediatric Emergency Procedures - Page 659, Christopher King,


Fred M. Henretig,2nd e

Gingival Abscess is usually common in primary teeth because More accessory canals are
present on buccal surface with porous floor & thin bony surface.Primary teeth have more
accessory canals in the inter-radicular area,which lead to formation of inter-radicular pus
drawing through gingival,preferably from buccal side due to thin bone.

13.In salivary reductase test of a child patient,slightly conductive caries gives what colour?

a.Red

b.Pink

c.Orchid

d.Green

Ans: c.Orchid

Refer: Principles and Practice Of Pedodontics,3rd e - Page 195

SALIVARY REDUCTASE TEST (SUSCEPTIBILITY TEST)

Principle involved: Measures the activity of the reductase enzyme present in salivary
bacteria, using a dye Diazoresorcinol.

Saliva is collected in a plastic container. The sample is then mixed with the dye.The caries
conduciveness is measured by color change, seen after 15 minutes. (A kit is availably under
the trade name Treatex.)
Interpretation: The evaluation is based on the color change

Color Time Score Caries activity

Blue 15 min. 1 Non conducive

Orchid 15 min. 2 Slightly conducive

Red 15 min. 3 Moderately conducive

Red Immediately 4 Highly conducive

Pink or White Immediately 5 Extremely conducive

Advantages: Quick results, as no incubation period is required.

Disadvantage: Test results vary with time after food intake and after brushing.

14.Flaring in the primary teeth treatment is?

a.Removable appliance

b.Fixed treatment

c.Consult orthodontist

d.Wait and watch

Ans: d.Wait and watch

Refer: Modern Pediatric Dentistry ,1st e - Page 209, Vinay Kumar Srivastava
Flaring in the primary teeth treatment is wait and watch.

15.In case of nursing bottle caries lower anteriors are usually unaffected with caries.This
effect occurs due to :

a.They are inherently resistant to caries

b.Lots of salivary flow leads to removal of any deposition

c.These teeth are usually covered by tongue during drinking sweet liquid or milk.

d.These teeth has very smooth covering compare to other ones.

Ans: d.These teeth has very smooth covering compare to other ones.

Refer: Modern Pediatric Dentistry - Page 160, Vinay Kumar Srivastava,1st e

In case of nursing bottle caries lower anteriors are usually unaffected with caries.This effect
occurs due to these teeth has very smooth covering compare to other ones.
16.In deciduous teeth calcium hydroxide pulpotomy is contraindicated due to ?

a.Leads to external resorption of tooth.

b. Leads to internal resorption.

c.The entire pulp tissue is devitalized and fixed.

d.Form calcium barrier within pulp tissue.

Ans: b. Leads to internal resorption.

Refer: Fundamentals of pediatric dentistry - Page 389,3rd e

In deciduous teeth calcium hydroxide pulpotomy is contraindicated due to internal resorption.

Treatment Objectives for Primary Tooth Pulpectomy


(1) Maintain the tooth free of infection,
(2) Biomechanically cleanse and obturate the root canals,
(3) Promote physiologic root resorption, and
(4) Hold the space for the erupting permanent tooth.

Indications for Pulpectomy of Primary Teeth


(1) Cooperative patient,
(2) Teeth with poor chance of vital pulp treatment,
(3) Strategic importance for space maintenance,
(4) Absence of severe root resorption,
(5) Absence of surrounding bone loss from infection,
(6) Expectation of restorability,
(7) Pulpless primary teeth with sinus tracts,
(8) Pulpless primary teeth in hemophiliacs,
(9) Pulpless primary teeth next to the line of a palatal cleft,
(10) Pulpless primary teeth when space maintainers or continued supervision are not
feasible (handicapped or isolated children.

Contraindications for Primary Tooth Pulpectomy


1. Teeth with nonrestorable crowns,
2. Periradicular involvement extending to the permanent tooth bud,
3. Pathologic resorption of at least one-third of the root with a fistulous sinus tract,
4. Excessive internal resorption,
5. Extensive pulp floor opening into the bifurcation,
6. Systemic illness such: as congenital or rheumatic heart disease, hepatitis, leukemia, and
children on long-term corticosteroid therapy, or those who are immunocompromised,
7. Primary teeth with underlying dentigerous or follicular cysts.
17.A 6 year old patient came to dental clinic .On routine radiographic examination it shows a
supernumerary mesiodens between two central incisors what should be the treatment plan
for this condition?

a.Remain them as such and extract them if sign of infection occurs.

b.Extract them immediately without injury to permanent tooth.

c.wait to erupt it and extract only after that.

d.Wait till 12 years of age of child and extraxt after that .

Ans: b.Extract them immediately without injury to permanent tooth.

Refer:Pinkham, 4th e ,page no.626

Supernumerary teeth are considered as one of the most significant dental anomalies during
the primary and early mixed dentition stages. They are of great concern to the dentists and
parents because of the eruption, occlusal, and esthetic problems they can cause.
Supernumerary teeth occur more frequently in the permanent dentition but rarely in primary
dentition. Mesiodens is the most common type of supernumerary teeth but rarely seen in
lower arch. Early recognition and diagnosis of supernumerary teeth is important to prevent
further complications in permanent dentition. Extract them immediately without injury to
permanent tooth.

18.Which of the following is a method of aversive conditioning?

a.Handover mouth

b.Tell show and do

c.Voice control/video

d.Modelling

Ans: a.Handover mouth

Refer: A Textbook of Public Health Dentistry - Page 492,1st e CM Marya

Aversive conditioning

Aversive conditioning consist of the HOME (Hand over mouth exercise) method and physical
restraints.

HOME, the purpose of HOME is to gain attention of the child so that communication can be
achieved. HOME is usually indicated for a healthy child who can understand but exhibit
defiant or hysterical behavior during treatment. Usually applied for 3 to 6 year old children,
which usually by than can understand simple verbal commands.

HOME should not be used for children under 3 years of age, handicapped, immature and
frightened children.
Before using this method, the dentist should definitely obtain the consent of the parent
before carrying out the HOME exercise. Some parents get a shock when they suddenly see
the dentist placing his or her hand over their childs mouth. Their first thought is that the
dentist has lost his or her mind and that the dentist is not professional. So bear in mind this
is a legitimate and effective method used by dentist to control children.

Another part of aversive conditioning is Physical restraints. It might sound crazy or


overboard, but some children require physical restraint to ensure that the dentist can carry
out their jobs safely and effectively. This is the last resort used for handling uncooperative
children or handicapped children. It involves the restriction of movement of the childs head,
hands, feet or body.

Types of restraint for the body :

pedi wrap
papoose board

sheets

beanbag with straps

towels and tapes

Types of restraint for the head :

head positioner
mouth blocks

banded tongue blades

mouth props

19.A child born with micrognathia and retrognathia usually has?

a.OSA

b.Hypoventilation

c.Hypocarbia

d.Hyperventilation

Ans: a.OSA

Refer: Principles and Practice of Pediatric Sleep Medicine,1st e - Page 273, Stephen H.
Sheldon, Meir H. Kryger, Richard Ferber

Infants with severe anatomical abnormalities such as micrognathia or nasal obstruction have
increased upper airway resistance accompanied by marked increases in respiratory effort,
and therefore negative luminal pressure. These infants typically manifest loud snoring with
marked labored breathing and complete obstructions often terminating with an arousal.
Despite a brisk genioglossal EMG response to upper airway occlusion or increased
resistance during sleep infants have an immediate and sustained decline in minute
ventilation . Infants with OSA have a decreased activation of the major upper airway dilator
muscle compared with nonapneic control subjects in response to airway occlusion,
potentially impairing the ability to respond to airway closure . During spontaneous
obstructions in term infants, decreased respiratory effort has been measured at the start of
an obstructive event, followed by a gradual increase in effort above baseline by the third
breath . By contrast, preterm infants have decreased respiratory effort throughout a
spontaneous obstructive event . Thus, term infants frequently demonstrate a robust
neuromuscular reflex activation of the upper airway during obstruction, which may be absent
in preterm infants. Mild dysmorphia predisposing toward OSA may be observed in infants
presenting with ALTEs including micrognathia/retrognathia and elongated uvula .
Thoracoabdominal asynchrony, suggestive of increased respiratory effort, has also been
reported in infants with ALTE . Infants that develop OSA are more likely to have a family
history of OSA and nasal allergies, indicating that immune function and craniofacial traits
may be risk factors for OSA early in life .

20.At what age ,is a child expected to have 12 permanent and 12 primary teeth?

a.6.5 years

b.7.5 years

c.9 year

d.11 year

Ans: c.9 year

Refer: shobha Tandon 1st ed 574

A child expected to have 12 permanent and 12 primary teeth at the age of 9 years .

21. Brushing technique recommended in preschool children?


a) Bass technique
b) Fones Technique
c) Horizontal scrub technique
d) Vertical
Ans: b) Fones Technique

Refer: Principles and Practice Of Pedodontics - Page 237,3rd e,Arthi rao

The oldest toothbrushing method was described in 1913 by Fones and is recommended
mainly for children.The Bass technique places emphasis on the removal of plaque from the
area above and just below the gingival margin. It was changed to the Modified Bass where
the bristle position and predominantly horizontal brush movements in the Bass method are
retained, but vertical and sweeping motions to create circles are added.The Stillman
technique is similar to the Bass technique. The vertical motions of the Stillman technique
may be combined with the Bass, as prescribed for the Modified Bass. Charters suggested
angling the brush head at 45 coronaly to the margin rather than apically. A 'vibratory' and
'slight rotary' movement is then applied before moving to the next group of teeth. On the
other hand, the Scrub and Fones techniques were more frequently recommended for
children. The more technically simple Scrub and Fones techniques are advocated for
children, with the more complex Bass and Modified Bass advocated for adults.

22.A fracture involving enamel, dentine and pulp in 52, is included in which Ellis fracture
classification?

a.Class III

b.Class-IV

c. Class-VI

d. Class-IX

Ans: d. Class-IX

Refer: Nikhil Marwah 3rd e /page no 749

Classification by Ellis and Davey (1960)


This classification is based on numeric system. Though it is simple, is only applicable to the
permanent dentition.
All the primary teeth have been grouped but detailed description has not been given
(Grouped as class 9).
It is one of the most widely accepted methods of classification.
Class I - Simple fracture of the crown involving little (or) no dentin.
Class II - Extensive fracture of the crown involving considerable dentin, but not the dental
pulp.
Class III - Extensive fracture of the crown involving considerable dentin and exposing the
dental pulp.
Class IV - The traumatized teeth that become nonvital with (or) without loss of crown
structure.
Class V - Teeth lost as a result of trauma.
Class VI - Fracture of the root with or without a loss of crown structure.
Class VII - Displacement of a tooth without fracture of crown (or) root.
Class VIII - Fracture of crown en masse and its replacement.
Class IX - Injuries to primary dentition
23. 2 %sodium fluoride application is done on a tooth root when extra oral time of avulsed
tooth is more than 60 min ,reason to this is?

a.Promote revascularisation

b.To prevent root caries

c.Prevent sensitivity

d.To prevent external resorption & delay ankylosis

Ans: d.To prevent external resorption & delay ankylosis

Refer: shobha Tandon 1st ed 357,520

To slow down osseous replacement of root surface ,treatment with 2 % NAF of the root
surface for 20 min is recommended.

24.Till what age new born child is called neonate?

a.8 weeks

b.4 weeks

c.6 weeks

d.18 weeks

Ans: 4 weeks

Refer:McDonald 9th /page no 150

Till 4 weeks of age new born child is called neonate.


25.Which is not true during transition from deciduous to permanent teeth?

a.Reduce arch circumference due to use of leeway space.

b.Increase interarch canine width

c.Slight labial placing of lower anteriors

d.Mesial drifting of premolar after primary canine exfoliation

Ans: d.Mesial drifting of premolar after primary canine exfoliation

Refer:Profitt 4th e/page no.100

During transition from deciduous to permanent teeth;

a.Reduce arch circumference due to use of leeway space.

b.Increase interarch canine width

c.Slight labial placing of lower anteriors

d.Distal shift of the permanent canines when primary 1st molars are exfoliated.

26.Which of the following is not a feature of normal mature swallow?

a.There is no abnormal forward tongue thrusting

b.There is slight contraction of perioral muscles

c.There is momentary contact of teet during swallowing

d.Tongue touch anterior part of palate behind the maxillary anterior teeth

Ans: d.Tongue touch anterior part of palate behind the maxillary anterior teeth

Refer:Nikhil Marwah 3rd e /page no 367

Characteristics of mature swallow: (Moyers)

Teeth are in occlusion

Mandible stabilized by contractions of mandibular elevators

Tongue tip held against the palate, above and behind maxillary incisors

Minimal lip contractions

27. Latest Terminology for following type of caries gaining almost acceptance?

a.Rapidly progressive
b.Severe early childhood caries

c.Nursing Bottle caries

d.Juvenile caries

Ans: b.Severe early childhood caries

Refer: Pediatric Dentistry: A Clinical Approach - Page 65, Goran Koch, Sven
Poulsen,2nd e

Early Childhood Caries is defined as the presence of one or more decayed (non-cavitated or
cavitated lesions), missing (due to caries) or filled tooth surfaces in any primary tooth in a
preschool-age child between birth and 71 months of age. The term "Severe Early Childhood
Caries" refers to "atypical" or "progressive" or "acute" or "rampant" patterns of dental caries.

28.What should be the minimum age of a child at which root of mandibular central incisor are
completed?

a.7.5 years

b.9 years

c.11 years

d.13 years

Ans:9 years
Refer: Pediatric Dentistry: A Clinical Approach - Page 2, Goran Koch, Sven Poulsen ,
2nd e

29.A 2 year old patient reported maxillary with his 2 central incisors intruded, having
maxillary fallen from bed 12 hours ago.What should the management plan in this patient?

a.Surgically reposition the teeth and splint

b.orthodontically reposition the teeth and splint

c.Wait for teeth, to erupt spontaneously

d.Reduce the opposing teeth

Ans: c.Wait for teeth, to erupt spontaneously

Refer:Mc Donalds 8th e, page no 479

Displacement of primary teeth (Luxation)

Lateral luxation is defined as displacement of a tooth in a direction other than axially. The
tooth may be displaced in a labial, lingual, or lateral direction. Damage to the periodontal
ligament and contusion or fracture of the supporting alveolar bone may accompany this
injury. Clinical examination reveals a tooth that is displaced in a lateral, palatal or lingual
direction and may be locked into its new position thus not mobile. The tooth usually is not
tender to touch. Pulp sensibility testing will likely give negative results. In immature teeth,
pulpal revascularization usually occurs. Radiographic findings reveal an increase in the
periodontal ligament space and displacement of the apex toward or through the labial bone
plate. Treatment consists of:

Primary teeth: Treatment depends on the degree of displacement, occlusal


interferences and time to exfoliation. The teeth may be allowed to passively
reposition if not interfering with occlusion. If interferences are present the tooth is
actively repositioned and splinted to the adjacent teeth for one to two weeks to allow
for healing. Primary teeth requiring positioning have an increased risk of developing
pulp necrosis compared to teeth that are left to spontaneously reposition. When the
injury is severe, the tooth is nearing exfoliation or the patient is uncooperative,
extraction should be considered.
Permanent teeth: Active repositioning of the tooth into its anatomically correct
position should be initiated as soon as possible. The tooth is repositioned using
finger pressure under local anesthesia. The tooth may need to be extruded to free
the apical lock in the cortical bone. If the tooth is displaced greater than 5mm, the
pulp is extirpated within 48 hours and the canal filled with calcium hydroxide. The
tooth is splinted to the adjacent teeth for two to four weeks. Antibiotics, tetanus
prophylaxis, and 0.2% chlorhexidine gluconate mouthrinse is prescribed.

Lateral luxations always have a dento-alveolar fracture component and the alveolar bone is
repositioned into its correct position to maintain alveolar integrity. The bone may be
stabilized with a flexible splint or suture material for four weeks.

In the primary dentition follow up treatment is clinical observation at two to three weeks and
clinical observation and radiographs at six to eight weeks and one year. In the permanent
dentition follow up is clinical and radiographic examination every two weeks while the splint
is in place and then six to eight weeks, six months and annually up to five years. There is
considerable risk for pulp necrosis and root resorption.

Intrusion is defined as apical displacement of the tooth into the alveolar bone. It is
accompanied by compression of the periodontal ligament, disruption of the neurovascular
supply to the pulp, contusion of the cementum and crushing fracture of the alveolar socket.
In severe injuries the tooth may be locked into the bone. Clinical findings reveals a tooth
that may appear shortened or even missing. In primary teeth the tooth apex is usually
displaced labially toward or through the labial bone plate. In permanent teeth the
displacement is into the alveolar bone. There is no tooth mobility nor tenderness to touch.
Radiographic findings reveal that the tooth is displaced apically and the periodontal ligament
space is not continuous. Determination of the position of the primary tooth in relationship to
the developing permanent may be determined by a lateral radiograph. Alternatively, if the
apex is displaced labially, the apical tip can be seen radiographically with the tooth
appearing shorter than its contralateral. If the apex is displaced palatally towards the
developing permanent tooth, the apical tip cannot be seen radiographically and the tooth
appears elongated. Treatment consists of:

Primary teeth: Allow the intruded tooth to spontaneously erupt unless radiographs
indicate intrusion into the developing tooth. The authors experience has been to
measure the amount of tooth exposed beyond the gingival margin. The tooth is
measured four weeks later. If any re-eruption has occurred another measurement is
taken four weeks later. This is repeated until the tooth is fully re-erupted (is even with
the contralateral tooth). If the tooth exhibits no evidence of re-eruption after a four
week period, extraction of the tooth is recommended to avoid ankylosis and possible
injury to the developing permanent tooth.

30.In a 5 year old patient 2nd molar is grossly carious and has to be extracted what is most
appropriate for this patient?

a.Band and loop space maintainer

b.Distal shoe appliance


c.No need to provide any such treatment

d.Extraction of 2nd molar and observation

Ans: b.Distal shoe appliance

Refer:McDonald 9th e , page no 563

Early loss of lower 2nd primary molars in conjunction with the first molar eruption results in
loss of spce.In such case , to avoid space loss distal shoe appliance is most appropriate.

31.What is the most appropriate restoratiove treatment for 5 year old child having multiple
carious lesions on multiple surfaces of molar teeth?

a.GIC

b.Composite

c.Amalgam

d.SS crown

Ans: d.SS crown

Refer:Pinkham, 4th e /page no.358

Stainless steel crown are indicated in primary teeth having multiple carious lesions.

32.if a child IQ level is measured 50,he/she will catergorized into?

a.Superior

b.Average

c.Mentally retarted

d.Intelligent

Ans: c.Mentally retarted

Refer:McDonalds, 8th e /page no 69

An intelligence test score that is obtained by dividing mental age, which reflects the age-
graded level of performance as derived from population norms, by chronological age and
multiplying by 100: a score of 100 thus indicates a performance at exactly the normal level
for that age group.

IQ Classifications in Educational Use

Wechsler, David. Wechsler Adult Intelligence Scale-Third edition


Psychological Corporation, 1997
Classification IQ Score Percent Included
Theoretical
Actual Sample
Normal Curve
Very Superior 130 and above 2.2 2.1
Superior 120-129 6.7 8.3
High Average* 110-119 16.1 16.1
Average 90-109 50.0 50.3
Low Average* 80-89 16.1 14.8
Borderline 70-79 6.7 6.5
Extremely Low* ** 69 and below 2.2 1.9

33.Best medium for avulsed permanent tooth in a peripheral village school?

a.Milk

b.Saliva/ice

c.Cold water

d.Handker chief

Ans: a.Milk

Refer:Mc Donald, 9th e /Page no.430

Best medium for avulsed permanent tooth in a peripheral village school milk. Milk remains
the most convenient, cheapest and readily available solution in most situations while also
being capable of keeping PDL cells alive. Hence, milk remains the storage medium of choice
for avulsed teeth that cannot be replanted immediately or very soon after the avulsion.

34.Babinksis sign in children include?

a.Extension of thumb of leg when tap/touch just below the end of fingers.

b.Flexion of one side off hand and leg when head turn another side

c.Upward extension of toes when sole of foot is touch firmly outer side

d.Flexion of toe and hyperextension of digits when clinician runs his finger on the outer side
planter surface of the foot from sole towards the thumb.

Ans: c.Upward extension of toes when sole of foot is touch firmly outer side

Refer:Nikhil Marwah ,3rd e/page no.99


Babinski reflex is one of the normal reflexes in infants. Reflexes are responses that occur
when the body receives a certain stimulus.

The Babinski reflex occurs after the sole of the foot has been firmly stroked. The big toe then
moves upward or toward the top surface of the foot. The other toes fan out.

This reflex is normal in children up to 2 years old. It disappears as the child gets older. It may
disappear as early as 12 months.

When the Babinski reflex is present in a child older than 2 years or in an adult, it is often a
sign of a brain or nervous system disorder. Disorders may include:

Amyotrophic lateral sclerosis (Lou Gehrig disease)


Brain tumor or injury

Meningitis

Multiple sclerosis

Spinal cord injury, defect, or tumor

Stroke

35.A Child with carious deciduous mandibular teeth has sensitivity to cold and sweet food
which subsides on removing on removing the stimulus .The diagram and most appropriate
treatment in this condition is?

a.Reversible pulpitis with direct pulp capping

b. Reversible pulpitis with pulpotomy

c. Reversible pulpitis with extraction


d.Irreversible pulpitis

Ans: a.Reversible pulpitis with direct pulp capping

Refer: Principles and Practice Of Pedodontics - Page 506, Arathi Rao,1st e

This treatment is only recommended when a small traumatic exposure occurs, during cavity
preparation of a vital non-infected pulp. A calcium hydroxide dressing is placed directly over
the pulp, followed by a lining and restoration, and the whole technique is carried out using
local anaesthesia and with adequate isolation from salivary contamination. It has been
suggested that the high cellular content of primary pulp tissue may be responsible for the
failure of direct pulp capping in primary teeth. Undifferentiated mesenchymal cells may
differentiate into osteoclastic cells in response to either the caries or direct pulp capping
which leads to internal resorption. It is also suggested that exposures on axial walls have a
poor prognosis as the pulp coronal to the exposure may be deprived of its blood supply and
undergo necrosis.

36.Which is not true during transition from deciduous to permanent teeth?

a.Reduce arch circumference due to use of leeway space by molar.

b.Increase interarch canine width

c.Slight labial placing of lower anteriors

d.Mesial drifting of premolar after primary canine exfoliation

Ans: d.Mesial drifting of premolar after primary canine exfoliation

Refer:McDonald 9th e,page no.551

37.During transition from primary to permanent dentition the additional space to align
incisors,after the period of mild normal crowding , is derived from 3 sources:

a.A slight increase in width across canines

b.Slight labial positioning of central and lateral incisors

c.A distal shift of permanent canines when primary molars are exfoliated.

In following mixed dentition analysis dista surface of both 2nd molar of primary teeth shows?
a.Edge to edge

b.Mesial step

c.Distal step

d.Class I malocclusion

Ans: b.Mesial step

Refer:McDonald 8th e/page no.591

Occlusal relationships in the mixed dentition parallel those in the permanent dentition, but the
descriptive terms are somewhat different. A normal relationship of the primary molar teeth is
the flush terminal plane relationship illustrated in Fig. 3-45. The primary dentition equivalent
of Angle's Class II is the distal step. A mesial step relationship correspond's to Angle's Class I.
An equivalent of Class III is almost never seen in the primary dentition because of the normal
pattern of craniofacial growth in which the mandible lags behind the maxilla.
At the time the primary second molars are lost, both the maxillary and mandibular molars
tend to shift mesially into the leeway space, but the mandibular molar normally moves
mesially more than its maxillary counterpart. This differential movement contributes to the
normal transition from a flush terminal plane relationship in the mixed dentition to a Class I
relationship in the permanent dentition.

Differential growth of the mandible relative to the maxilla is also an important contributor to
the molar transition. As we have discussed, a characteristic of the growth pattern at this age is
more growth of the mandible than the maxilla, so that a relatively deficient mandible
gradually catches up. Conceptually, one can imagine that the upper and lower teeth are
mounted on moving platforms, and that the platform on which the lower teeth are mounted
moves a bit faster than the upper platform. This differential growth of the jaws carries the
mandible slightly forward relative to the maxilla during the mixed dentition.

If a child has a flush terminal plane molar relationship early in the mixed dentition, about 3.5
mm of movement of the lower molar forward relative to the upper molar is required for a
smooth transition to a Class I molar relationship in the permanent dentition. About half of this
distance must be supplied by differential growth of the lower jaw, carrying the lower molar
with it. The other half can be obtained from the leeway space, which allows greater mesial
movement of the mandibular than the maxillary molar.

Only a modest change in molar relationship can be produced by this combination of


differential growth of the jaws and differential forward movement of the lower molar. It must
be kept in mind that the changes described here are those that happen to a child experiencing
a normal growth pattern. There is no guarantee in any given individual that differential
forward growth of the mandible will occur, nor that the leeway space will close in a way that
moves the lower molar relatively forward.

The possibilities for the transition in molar relationship from the mixed to the early
permanent dentition are summarized in Fig. 3-45. Note that the transition is usually
accompanied by a one-half cusp (3 to 4 mm) relative forward movement of the lower molar,
accomplished by a combination of differential growth and tooth movement. A child's initial
distal step relationship may change during the transition to an end-to-end (one-half cusp
Class II) relationship in the permanent dentition, but is not likely to be corrected all the way
to a Class I. It is also possible that the pattern of growth will not lead to greater prominence
of the mandible, in which case the molar relationship in the permanent dentition probably
will remain a full cusp Class II.

Similarly, a flush terminal plane relationship, which produces an end-to-end relationship of


the permanent molars when they first erupt, can change to Class I in the permanent dentition,
but can remain end-to-end in the permanent dentition if the growth pattern is not favorable.

Finally, a child who has experienced early mandibular growth may have a mesial step
relationship in the primary molars, producing a Class I molar relationship at an early age. It is
quite possible for this mesial step relationship to progress to a half-cusp Class III during the
molar transition with continued mandibular growth. On the other hand, if differential
mandibular growth no longer occurs, the mesial step relationship at an early age may simply
become a Class I relationship later.

For any given child, the odds are that the normal growth pattern will prevail, and that there
will be a one-half cusp transition in the molar relationship at the time the second primary
molars are lost. It must be understood that although this is the most likely outcome, it is by no
means the only one. The possibility that a distal step will become Class II malocclusion or
that a flush terminal plane will become end-to-end is very real. Class III malocclusion is
much less common than Class II, but a child who has a mesial step relationship at an early
age is also at some risk of developing Class III malocclusion as time passes.
ORAL SURGERY

38.In 1st picture is shown and in 2nd picture post-operative radiograph is shown what is
procedure known as?

1st

2nd

a.Genioplasty

b.Sub apical osteotomy

c.BSSO

d.Chin Reduction

Ans: a.Genioplasty
Refer: Vinod Kapoor 2nd ed 542

The specific medical terms mentoplasty and genioplasty are used to refer to the reduction
and addition of material to a patient's chin. This can take the form of chin height reduction or
chin rounding by osteotomy, or chin augmentation using implants.

39.During tooth extraction rotation movement used for?

a.Maxillary central incisor

b.Maxillary 1st premolar

c.Maxillary canine

d.Mandibular central incisor

Ans: a.Maxillary central incisor

Refer: Contemporary Oral and Maxillofacial Surgery - Page 147, Larry J. Peterson,4th e

During tooth extraction rotation movement used for Maxillary central incisor.Rotation is used
for extraction of tooth with conical roots.Technique of using a luxator, then apical pressure,
and then rotation can be used on all maxillary teeth from the central incisor to the second
bicuspid.

40.Ideal method of intubation in a patient with bilateral TMJ ankylosis and retruted mandible
is?

a.Direct larygoscopy

b.Conventional laryngoscopy

c.Tracheostomy

d.Video laryngoscopy

Ans: d.Video laryngoscopy

Refer: Atlas of Oral and Maxillofacial Surgery - Page 924,/1st e

Ideal method of intubation in a patient with bilateral TMJ ankylosis and retruted mandible is
Video laryngoscopy.

41.Which of the following are used to check the efficiency of sterliation?

a.Bacteria

b.Virus

c.Spore Former

d.Fungus
Ans: a.Bacteria

Refer: Contemporary Oral and Maxillofacial Surgery Page 67, Larry J. Peterson,4th e

Transmissible agents (such as spores, bacteria and viruses) can be eliminated through
sterilisation. This is different from disinfection, where only organisms that can cause disease
are removed. The efficacy of sterilization was evaluated by comparison of bacterial growth
obtained in monitoring by biological indicators and swab test method

42.Investigations to be done in a case of facial symmetry before orthognathic surgery


performed?

a.CT Scan

b.Bone scan

c.COGS

d.Grummons analysis

Ans: d.Grummons analysis

Refer: Textbook of Oral Radiology - Page 263, Ghom,2nd e


Grummons analysis: Investigation procedure to be done in a case of facial symmetry before
orthognathic surgery.

43.In Given OPG, what will be your diagnosis:

a.Osteoma

b.Odontoma

c.Ameloblastoma/Dentigerous cyst

d.Fibrous Dysplasia
Ans: a.Osteoma

Refer: Textbook of Oral and Maxillofacial Surgery - Page 559, By Neelima Anil Malik,3rd
e

Osteomas of the facial bones are a rare entity and very few cases have been reported in the
literature. Osteomas are benign neoplasms, often asymptomatic and consist of well-
differentiated matured bone. There are three varieties of osteomas- the central type arising
from the endosteum, the peripheral type arising from the periosteum, and the extra-skeletal
soft tissue osteomas which usually develops within the muscle.

44.Important role of vitamin K is?

a.Production of factor 2

b.Production of factor 7

c.Production of stuart factor

d.Inhibit catalysing of prothrombin

Ans: d.Inhibit catalysing of prothrombin

Refer: Shafer 7th ed 652/6th ed 650

Prothrombin is produced in the liver and is co-translationally modified in a vitamin K-


dependent reaction that converts ten glutamic acids on prothrombin to gamma-
carboxyglutamic acid (Gla). In the presence of calcium, the Gla residues promote the binding
of prothrombin to phospholipid bilayers. Deficiency of vitamin K or administration of the
anticoagulant warfarin inhibits the production of gamma-carboxyglutamic acid residues,
slowing the activation of the coagulation cascade.

45.Important role of vitamin K is Inhibit catalysing of prothrombin.

Circumzygomatic suspension is done for?

a.Lefort I

b.Lefort II

c. Lefort III

d.Zygomatic fractures

Ans: a.Lefort I

Refer: Contemporary Oral and Maxillofacial Surgery Page 480, Larry J. Peterson,4th e
Circumzygomatic suspension is done for Lefort I. Midfacial ( Le Fort) fractures are types of
facial fractures involving the maxillary bone and surrounding structures in a usually bilateral
and pyramidal. Disimpaction of the midface segments was performed manually with Rowe
disimpaction forceps and circumzygomatic wire suspension technique was used as method
of treatment.

46.Which of the following is the best approach to Nao-orbital-ethmoidal fracture?

a.Open sky approach

b.Coronal approach

c.Seagull incision

d.H incision

Ans: b.Coronal approach

Refer: Contemporary Oral and Maxillofacial Surgery Page 838, Larry J. Peterson,4th e

Coronal approach is the best approach to Nao-orbital-ethmoidal fracture. The bony naso-
orbital-ethmoid (NOE) complex is a 3-dimensional delicate anatomic structure. Damages to
this region may result in severe facial dysfunction and malformation. The management and
optimal surgical treatment strategies of NOE fractures remain controversial. For a patient
with NOE trauma, doctors should perform comprehensive clinical examination and
radiographic analysis to assess the type and extent of fracture.

47.Masticatory space infection is usually due to?

a.Mandibular molar

b.Parotid space infection

c.Lateral pharyngeal infection

d.Pteryomandibular space infection

Ans: a.Mandibular molar

Refer: Oral and Maxillofacial Surgery - Page 511, Lars Andersson, Karl-Erik Kahnberg,
M. Anthony Pogrel,1st e

Submasseteric abscesses are relatively rare, and may be confused with a parotid abscess
or parotitis. They tend to be chronic. The submasseteric space may be involved by infections
that spread from the buccal space. Sometimes mandibular fractures in the region of the
angle of the mandible may cause an infection of the submasseteric space. The signs and
symptoms of a submasseteric abscess may include marked trismus (i.e. difficulty opening
the mouth, since the masseter elevates the mandible and it becomes restricted) and swelling
in the region of the masseter muscle. The treatment of a submasseteric space infection is
usually by surgical incision and drainage, and the incision is placed intra-orally (inside the
mouth) or both intra and extra-orally if other parts of the masticator space are involved.The
submasseteric space is sometimes involved by the spread of odontogenic infections, such
as a pericoronal abscess associated with an impacted mandibular third molar (lower wisdom
tooth) when the apices of the tooth lie very close to or within the space

48.Which type of endotracheal tube (ETT) is preferred for submental approach in complex
NOE and zygomatic fracture?

a.Metallic ETT

b.Flexible silicone ETT

c.PVC ETT

d.pre-curved metallic ETT

Ans: b.Flexible silicone ETT

Refer: Oral and Maxillofacial Surgery - Page 871, Lars Andersson, Karl-Erik Kahnberg,
M. Anthony Pogrel,1st e

Flexible silicone ETT is preferred for submental approach in complex NOE and zygomatic
fracture. The submental approach for intubation allows an unhindered reduction and fixation
of the complex maxillofacial fractures in which simultaneous access to nasal pyramid
fractures is required and thereby avoiding the need for tracheostomy. Flexometallic ETT is
preferred since it is reinforced with a metallic spring material which has a shape memory and
thus the tube is flexible, kink-resistant and retains patency despite the acute angle of the
airway, particularly at the submental route. Initially standard orotracheal intubation was
carried out and ETT is connected to the breathing circuit. The orotracheal intubation is then
transformed into the submental intubation using the following surgical technique. A standard
extubation procedure was carried out after completion of the anticipated procedure.

49.In basal skull fracture which approach is not used?

a.Nasal intubation

b.Oral intubation

c.Submental intubation

d.Retromolar intubation

Ans: a.Nasal intubation

Refer: Atlas of Oral and Maxillofacial Surgery - Page 1453, Deepak Kademani, Paul
Tiwana,1st e

Submental intubation technique consists of passing the tube through the anterior floor of
mouth, allowing free intraoperative access to oral cavity and nasal pyramid without
endangering patients with skull base trauma. Oral intubation,Submental intubation and
Retromolar intubation are all used for basal skull fracture.

50.Internal opening of brachial fistula is at?

a.Posterior pharyngeal wall /palatoglossal arch

b.Fossa of tonsil

c.Second premolar area

d.Third molar area

Ans: b.Fossa of tonsil

Refer: Stell & Maran's Textbook of Head and Neck Surgery and Oncology, Fifth
Edition,page no 222

The branchial fistula arising from each arch can be identified from the position of the internal
and external openings. Branchial fistulae are formed due to the abnormal persistence of the
embryonic second branchial cleft. Branchial fistulae arising from second and third arches are
common than from first and fourth arches. The tract in second arch fistula extends deep to
the platysma, along the carotid sheath, passing between the bifurcation of the carotid
arteries after crossing over the hypoglossal and glossopharyngeal nerves and passes below
the stylohyoid ligament. It opens internally in the lateral wall of the pharynx region of the
tonsillar fossa.

51.A young girl came to the hospital with severe swelling and redness on affected areas .On
intra oral examination ,24 was severely infected with signs of acute alveolar abscess
.Cellulitis occurs in this patient most probably due to which space infection?

a.Canine space

b.Buccal space

c.Masseteric space infection

d.Cavernous sinus infection


Ans: b.Buccal space

Refer: Oral and Maxillofacial Surgery - Page 506, Lars Andersson, Karl-Erik Kahnberg,
M. Anthony Pogrel

The buccal space (also termed the buccinator space) is a fascial space of the head and
neck (sometimes also termed fascial spaces or tissue spaces). It is a potential space in the
cheek, and is paired on each side. The buccal space is superficial to the buccinator muscle
and deep to the platysma muscle and the skin. The buccal space is part of the subcutaneous
space, which is continuous from head to toe.

The boundaries of each buccal space are:

the angle of the mouth anteriorly,


the masseter muscle posteriorly,

the zygomatic process of the maxilla and the zygomaticus muscles superiorly,

the depressor anguli oris muscle and the attachment of the deep fascia to the
mandible inferiorly,

the buccinator muscle medially (the buccal space is superficial to the buccinator),[1]

the platysma muscle, subcutaneous tissue and skin laterally (the space is deep to
platysma),[1]

Communications
to the pterygomandibular space, infratemporal space, submasseteric space or even
to the lateral pharyngeal space posteriorly,
to the infraorbital (canine) space superiorly,

52.Quantitative radiographic bone density is achieved in socket after how many months of
tooth extraction?

a.1 month

b.2 month

c.6 month

d.12 month

Ans: c.6 month

Refer: Contemporary Oral and Maxillofacial Surgery Page 198, Larry J. Peterson,4th e

Quantitative radiographic bone density is achieved in socket after 6 months of tooth


extraction. Computed tomography (CT) measurements were taken at baseline and 6
months. After 6 months of healing, nal CT measurements were performed, and trephine
core biopsy specimens were obtained for histologic analysis. Implants were placed
immediately after biopsy harvesting.

53.Sutures taken for closing oral-antral fistula should be removed after?

a.Should not be removed until it get infected.

b.Should be removed in 10 days.

c.Removed in 10 days

d.Removed in 3 days

Ans: b.Should be removed in 10 days.

Refer: Contemporary Oral and Maxillofacial Surgery Page 425, Larry J. Peterson,4th e

Sutures taken for closing oral-antral fistula should be removed after 10 days. At 10 days from
surgery the suture was removed and the mucosa appeared healthy.

54.A patient having painless hard swelling over the left ramus area (CT given)is most
probably ?

a.Parotid swelling/pleomorphic adenoma

b.Osteoma

c.Odontoma

d.Osteochondroma
Ans: b.Osteoma

Refer: Maxillofacial Imaging - Page 68, By Tore A. Larheim, Per-Lennart A.


Westesson/1st e
painless hard swelling over the left ramus area (CT given)is most probably Osteoma

55.Cracked pot sound is present in which fracture?

a.Lefort1,2,3

b.Guerin fracture

c.Suprazygomatic fracture

d.Pyramidal fracture

Ans: b.Guerin fracture

Refer: Textbook of Oral and Maxillofacial Surgery - Page 609, By Balaji,3rd e

Lefort I - Slight swelling of the upper lip, ecchymosis is present in the buccal sulcus
beneath each zygomatic arch, malocclusion, mobility of teeth. Impacted type of
fractures may be almost immobile and it is only by grasping the maxillary teeth and
applying a little firm pressure that a characteristic grate can be felt which is diagnostic
of the fracture. Percussion of upper teeth results in cracked pot sound. Gurin's sign
is present characterised by ecchymosis in the region of greater palatine vessels.

56.Evaluate the following OPG carefully.waht is the treatment should be given in this case?

a.2 miniplates at symphysis anteriorly and one on angle

b.1 miniplate on symphysis region on 2 on angle

c.Dynamic compression plates on both sides

d.2 plates at anterior and 2 over posterior fracture.


Ans: a.2 miniplates at symphysis anteriorly and one on angle

Refer: Atlas of Operative Oral and Maxillofacial Surgery


edited by Christopher J. Haggerty, Robert M. Laughlin,1st e ,page no 107

This is clearly according to champys principle of internal fixation of mandibular fracture,for a


functionally stable fixation.Wwhen using anything less stable then reconstruction plate, two
points of fixation should be applied.

57.Position of extraction forceps during replantation of teeth?

a.At CEJ

b.Coronal to CEJ

c.At root surface

d.Subgingival irrespective of CEJ

Ans: b.Coronal to CEJ

Refer: Contemporary Oral and Maxillofacial Surgery Page 126, Larry J. Peterson,4th e

Position of extraction forceps during replantation of teeth Coronal to CEJ.

58.The following high definition CT Shows:

a.Displace subcondylar fracture

b.Displaced and dislocated subcondylar fracture

c.Ramus fracture

d.Undisplaced subcondylar fracture


Ans: d.Undisplaced subcondylar fracture

Refer: Contemporary Oral and Maxillofacial Surgery Page 410, Larry J. Peterson,4th e

Non displaced condylar fracture:a linear fracture with the proximal fragment retaining its
usual anatomic relationship with the distal fragment.

59.What is the treatment of choice for a communited symphyseal fracture with undisplaced
left condylar fracture?

a.Use of Champy mini plate/IMF

b.use of load shearing plate

c.Use of reconstruction load bearing plate

d.Dynamic compression plates

Ans: c.Use of reconstruction load bearing plate

Refer: Atlas of Operative Maxillofacial Trauma Surgery: Primary Repair of Facial and
orofacial complex,1st e
edited by Michael Perry, Simon Holmes,page no 124,1st e

Communited Fracture if not managed by closed reduction , needed a strong load bearing
fixation .Miniplate usually cannot hold the segments together.Dynamic compression plate is
too bulky for the symphyseal region.A recon plate can fix.

60.A patient who undergone TMJ surgery,all of the following can be administered to control
bradycardia except?
a.Atropine

b.Adrenaline

c.Glycopyyrolate

d.Phenylephrine

Ans: d.Phenylephrine

Refer: Clinical Anesthesia - Page 351, Paul G. Barash,1st e

Following can be administered to control bradycardia:

a.Atropine

b.Adrenaline

c.Glycopyyrolate

Reflex Bradycardia is a known effect of phenylephrine. Phenylephrine often causes a reflex


bradycardia. The vessels vasoconstrict (due to alpha-2 receptors), and blood pressure goes
up. The baroreceptors monitor blood pressure, and when they see an increase in blood
pressure, they invoke a bradycardia in the heart as a compensatory measure. It is often the
drug of choice for hypotensive patients who are tachycardic, as other pressor choices
(dopamine, epinephrine, norepinephrine) can increase heart rate more via beta-1
stimulation.

61.In PN/INR test , we assess the defect in ?

a.Extinsic and common pathway

b.Intrinsic and common pathway

c.Extrinsic only

d.Intrinsic only

Ans: c.Extrinsic only

Refer: Harrison's Principles of Internal Medicine - Page 419,17 th e

In PN/INR test , we assess the defect in Extrinsic pathway only. The prothrombin time
(PT) along with its derived measures of prothrombin ratio (PR) and international
normalized ratio (INR) are assays evaluating the extrinsic pathway of coagulation. This
test is also called "ProTime INR" and "PT/INR". They are used to determine the clotting
tendency of blood, in the measure of warfarin dosage, liver damage, and vitamin K status.
PT measures factors I (fibrinogen), II (prothrombin), V, VII, and X. It is used in conjunction
with the activated partial thromboplastin time (aPTT) which measures the intrinsic pathway
and common pathway.

62.What type of wiring is shown in the picture below?

a.Ivy loop wiring

b.Gilmer direct wiring

c.Essing wiring

d.IMF

Ans: b.Gilmer direct wiring

Refer: Textbook of Oral and Maxillofacial Surgery - Page 331, Neelima Anil Malik,
Malik,3rd e

GILMERS WIRING It is used for IMF.Most common and simple method.Few firm teeth
in the mandible as well as in maxilla are chosen.At least one firm teeth must be
chosen anterior and posterior to thefracture line.A pre stretched 20 cm long 26
guage wire is taken and passed aroundthe neck of the chosen tooth.Both the ends
of the wire are brought out on the buccal side andtwisted.

63.Hypermobility of TMJ is seen in all except?

a.Anterior dislocation of the disk

b.Marfans syndrome

c.Tardive dyskinesia

d.Phenothiazine therapy

Ans: a.Anterior dislocation of the disk

Refer: Oral and Maxillofacial Surgery - Page 1236, edited by Lars Andersson, Karl-Erik
Kahnberg, M. Anthony Pogrel/1st e

Hypermobility of TMJ is seen in:


Marfans syndrome

Tardive dyskinesia

Phenothiazine therapy

Anterior dislocation of the disk would reduce mobility of the condyle within TMJ joint.

64.Following picture of a child shows a device used for?

a.Treat fracture mandible

b.Treat obstructive sleep apnoea

c.Treat retrognathic mandible

d.To hold the neck.

Ans: b.Treat obstructive sleep apnoea

Refer: Clinical Review of Oral and Maxillofacial Surgery - Page 315, Shahrokh C.
Bagheri, Chris Jo,2nd e

This picture show bilateral mandibular distractors placed in the mandible .In child ,distraction
is done to correct obstructive sleep apnoea.

65.Bone plates should be remove in all of the following cases except:

a.Loosening hardware and sign of infection


b. Loosening hardware without sign of infection

c.Infection around the plate and loss of bone

d.Malpositioned plates

Ans: d.Malpositioned plates

Refer: Oral and Maxillofacial Surgery - Page 1247 , edited by Lars Andersson, Karl-
Erik Kahnberg, M. Anthony Pogrel/1st e

A loose hardware is a source of infection and should be removed as soon as


possible.Malpositioned plates , if not infected or loose, may be left in situ, if the bone reunion
has occurred and corrective osteotomy is not desired.

66.A 10 year old child came to your clinic with a complain of constricted maxilla and absent
left maxillary canine.Patient gives a history of repair of cleft lip when he was 3 months old
and cleft palate repair when he was 1 year old.Presently cleft of alveolus is present at left
maxillary canine area preventing eruption of canine,What will be the most appropriate
treatment for this condition?

a.Bone graft alveolus and surgical repositioning of canine

b.Extraction of canine

c.Expansion of arch first, then bone grafting with surgical exposing the canine

d.Wait till permanent canine erupt

Ans: c.Expansion of arch first, then bone grafting with surgical exposing the canine

Refer: Contemporary Oral and Maxillofacial Surgery Page 638, Larry J. Peterson,4 th
e

Alveolar bone grafting is an important part of the reconstructive journey for many cleft lip and
palate patients. The reconstruction of the alveolar cleft can provide both aesthetic and
functional benefits to the patient. To be able to effectively treat alveolar clefts, it is essential
to possess an understanding of several aspects of the problem. The initial operation is
performed at the age of three to four weeks without special prior treatment. The nasal floor
of the hard palate, in one layer, is closed with a vomerine mucosal flap and anterior to the
alveolar process by direct adaptation of the labial soft tissues in two layers. The vomerine
flap, which on the oral side is first covered with granulation tissue, has after some few weeks
a stable covering of secondary epithelium. At this junction, the orthopaedic correction of the
jaw is started and continued up to the age of six months. Special expansion plates are used,
fixed to a head cap by means of extra-oral shafts. At the second operation, the components
of the upper jaw should be ideally positioned in relation to each other and in correct
occlusion. Careful repair of the lip is now combined with transplantation of autografts, chips
and marrow, in the cleft in the hard palate and alveolar process. The donor site is tibia..A
continuous orthodontic control is subsequently kept until the permanent bite is fully
developed. At the third operation, which is usually performed at one year, the posterior
palate is closed.
67.What is best indicated to treat 1.5 x3x 3cm size ameloblastic lesion in a body of mandible
in otherwise healthy young male adult patient?

a.Radiotherapy must be considered after surgery

b.Surgical resection of lesion with including 2mm normal bony margin

c.Only curettage of lesion is sufficient

d.Surgical resection of lesion with including 10 mm normal bony margin.

Ans: b.Surgical resection of lesion with including 2mm normal bony margin

Refer : Contemporary Oral and Maxillofacial Surgery Page 585, Larry J. Peterson,4 th
e

Surgical resection of lesion with including 2mm normal bony margin is indicated to treat 1.5
x3x 3cm size ameloblastic lesion in a body of mandible in otherwise healthy young male
adult patient.

68.Which side of TMJ is not affected by ankylosis?

a.Right side

b.left side

c.Both

d.None

Ans: b.left side

Refer: Neelima Malik, page no 263,3rd e

Most prominent feature is unilateral TMJ ankylosis is flattening of face on the opposite side
and fullness of the face on the affected side.
PROSTHODONTICS

69.If an inlay wax is stored in a fridge,before investment procedure the wax pattern should
be?

a.Should never store in fridge under any circumstances

b.Used immediately for making pattern

c.Marginally heat,readapted and then wait for normal temperature.

d.Use only after achieving normal room temperature of wax

Ans: d.Use only after achieving normal room temperature of wax

Refer:Phillips , 11th e ,page no 287

Casting is the process by which a wax pattern of a restoration is converted to a replicate in


dental alloy. The casting process is used to make dental restorations such as inlays, onlays,
crowns, bridges, and removable partial dentures. Because castings must meet stringent
dimensional requirements, the casting process is extremely demanding. In dentistry, virtually
all casting is done using some form or adaptation of the lost-wax technique. The lost-wax
technique has been used for centuries, but its use in dentistry was not common until 1907,
when W.H. Taggart introduced his technique with the casting machine.
Casting can be defined as the act of forming an object in a mold .The object thus formed is
also called as a casting .
Objectives of casting
1) To heat the alloy as quickly as possible to a completely molten condition.
2) To prevent oxidation by heating the metal with awell adjusted torch .
3) To produce a casting with sharp details by having adequate pressure to the well melted
metal to force into the mold.

Dimensional Changes in the Lost-Wax Technique


If materials used during the casting process didn't shrink or expand, the size of the final cast
restoration would be the same as the original wax pattern. However, dimensional changes
occur in most of the steps and, in practice, the final restoration may not be exactly the same
size as the pattern. The management of these dimensional changes is complex, but can be
summarized by the equation:
wax shrinkage + metal shrinkage = wax expansion + setting expansion + hygroscopic
expansion + thermal expansion .This equation balances the shrinkage (left sideof equation)
against the expansion (right side ofequation) that occurs during the casting process. If the
final restoration is to fit the die, the shrinkage and expansion during the casting process
bmust be equal. Shrinkage forces in the casting process come from two sources: wax and
metal. Although the die restricts the wax from shrinking to a large degree while the pattern is
on the die, residual stresses may be incorporated into the pattern and released during
investing, when the pattern isremoved from the die. Furthermore, if the investingis done at a
temperature lower than that atwhich the wax pattern was formed, the wax willshrink
significantly because of the high coefficientof thermal expansion of waxes. Metal shrinkage
occurs when the moltenmetal solidifies, but this shrinkage is compensated by introducing
more metal as the casting solidifies. However, once the entire casting has reached the
solidus temperature of the alloy, shrinkage will occur as the casting cools to room
temperature. As for wax, the metallic shrinkage that occurs below the solidus is caused by
the coefficient of thermal expansion for the alloy. Cooling shrinkage may reach 2.5% for an
alloy that cools from a high solidus temperature (1300" to 1400' C), depending on the
coefficient of thermal expansion of the alloy. A typical shrinkage range for most alloys is
1.25% to 2.5%. Furthermore, because the casting is solid at this point, the only possible
compensation mechanismis to start with a void space that is 1.25% to2. 5% too large. Thus,
shrinkage of wax and metalmust be compensated with expansion in the investment if the
casting is to have the appropriate dimensions.

Expansion and shrinkage of inlay casting wax is very sensitive to temperature normally ,soft
wax shrinks more than hard wax .High shrinking wax cause significant pattern distortion
when it solidifies .

Accuracy of the Lost-Wax Technique


A casting should be as accurate as possible, although a tolerance of rt0.05% for an inlay
casting is acceptable. If the linear dimension of an average dental inlay casting is assumed
to be 4 mm, +0.05% of this value is equal to only +2ym, which suggests that if two castings
made for the same tooth have a variation of 4 ym, the difference may not be noticeable. To
visualize this dimension, recall that the thickness of an average human hair is about 40 ym.
Therefore the tolerance limits of a dental casting are approximately one-tenth the thickness
of a human hair. To obtain castings with such small tolerancelimits, rigid requirements must
be placed not only on the investment material but also on theimpression materials, waxes,
and die materials. Naturally, technical procedures and the proper handling of these materials
are equally important. The values for the setting, hygroscopic, and thermal expansions of
investment materials may vary from one product to another, and slightly different techniques
may be used with different investments. In each case, the values obtained for any one
property should be reproducible from one batch to another and from one casting to another.

70. Which of the following treatment approaches should be considered for a severely
extruded abutment tooth the loss of which would create a distal extension area?

a.Occlusal recontouring

b.Endodontic therapy and coping placement

c.Cast restoration

d.Nothing could be done so extract it and provide cantilever bridge

Ans: b.Endodontic therapy and coping placement

Refer: Textbook of Complete Denture Prosthodontics


By DL Sarandha, Sarandha D. L., Zakir Hussain, Uthkarsh,page no.152/1st e
We should try to prevent to form distal extension area because it always lead to complicate
the procedure of restroration either by fix or removable means.In cases of severly extruded,
our preference should be elective endodontics and coping placement to avoid the creation of
distal extension area.

71.Inherent bone remodelling around implant surface with in 1 year of implant placement ?

a.1-1.5mm

b.1.5-2mm

c. 2-3mm

d. 3-3.65mm

Ans: a.1-1.5mm

Refer:Rosentheil ,page no.186/4th e

Inherent bone remodelling around implant surface with in 1 year of implant placement 1-
1.5mm.The dimension of the peri-implant mucosa has been demonstrated to resemble that
of the gingiva at teeth and included a 2-mm-long epithelial portion and a connective tissue
portion about 11.5 mm long. The entire contact length between the implant and the
epithelial and the connective tissue portions is defined as the biological width.

72.While delivering a maxillary and mandibular partial denture a dentist noticed a gap
between teeth.What will be the most valid justification of this error?

a.Defective bite registration

b.At the time of bite registration patient co doesnot coincide Cr.

c.Uneven pressure applied by patient leading to movement of teeth during try-in

d.Due to processing error

Ans: a.Defective bite registration

Refer: Mccracken's Removable Partial Prosthodontics - Page 369/13th e

While delivering a maxillary and mandibular partial denture a dentist noticed a gap between
teeth is due to defective bite registration.

73.If more than one pattern has to be invested in a single casting ring then what should be
the minimum distance between two pattern?

a.3mm

b.4mm
c.2mm

d.5mm

Ans: a.3mm

Refer:Phillips,11th e,page no 341

If more than one pattern has to be invested in a single casting ring then the minimum
distance between two pattern is 3mm.If a several patterns are invested in the same ,they
should not be place too close together.

74.Primary indication of using double cord technique taking impression?

a.To prevent excessive tissue fluid seepage from gingival margin.

b.To prevent excessive haemorrhage from the gingival

c.T record the knife edge margin preparation as it is critical to record

d.When tooth preparation is at the level of gingival so to accurately expose it.

Ans: d.When tooth preparation is at the level of gingival so to accurately expose it

Refer: Phillips' Science of Dental Materials - Page 157,11th e

Double cord technique

This can be used when single or multiple teeth are prepared. It involves 2 cards, one placed
above the other. A thin 00 retraction cord is first packed to control the gingival seepage and
hemorrhage. The second large cord is impregnated with a hemostatic agent and placed
above the first cord for a minimum of 4 minutes and removed before the impression is made.
The advantage of this technique is that the first cord remains in place within the sulcus and
thus reduces the tendency of the gingival cuff to recoil and displace the impression material.
This approach not only helps control gingival hemorrhage and exudates but also prevents
the tearing of the sulcus impression because of inadequate bulk. Another advantage is that
the first cord protects the tearing of the gingival epithelium.

75.Biologic width around the implants :

a.3-3.5mm

b. 3-3.65mm

c. 4-4.5mm

d. 4-5mm

Ans: b. 3-3.65mm

Refer: Dental Implant Prosthetics - Page 967,2nd e , Carl E. Misch


Biologic width around the implants 3-3.65mm.

76.Biggest disadvantage of using two step impression technique in final impression is:

a.In ability to correctly reposition and seating the tray in the same position

b.Presence of air bubbles in impression

c.Leads to wastage of impression material

d.Excessive time consumption

Ans: a.In ability to correctly reposition and seating the tray in the same position

Refer: Textbook of Prosthodontics - Page 242, Nallaswamy,1st e


The disadvantages of this two-stage technique include the additional time of having to wait
for two materials to set, contamination of the putty with saliva which may prevent light body
adhering to it, and difficulty in reseating the set putty in the mouth. The most accurate
impression is usually achieved using heavy-body and light-body addition silicone in
conjunction with a rigid tray."

77.Normal t and d sound is created by?

a.Contact of the tongue with hard palate and lingual surface of incisors

b.Protusion of tongue between incisors

c.Spread of tongue between upper and lower posterior teeth

d.Forceful explusion of air between upper incisors & lips.

Ans: a.Contact of the tongue with hard palate and lingual surface of incisors

Refer: Removable Partial Dentures: A Clinician's Guide - Page 115, John D. Jones, Lily
T. Garcia,1st e

LINGUOALVEOLAR SOUNDS Formed with the valve formed by contact of the tip of the
tongue with the most anterior part of the palate (the alveolus) or the lingual sides of the
anterior teeth. T, D, S, Z, V & 1 are representative of the linguoalveolar group of sounds .
Normal t and d sound is created by Contact of the tongue with hard palate and lingual
surface of incisors

LINGUOALVEOLAR SOUNDS Sibilants (sharp sounds) s, z, sh, ch & j (with ch & j being
affricatives) are alveolar sounds, because the tongue and alveolus forms the controlling
valve. Important observations when these sounds are produced are the relationship of the
anterior teeth to each other.
78.A patient who recently got a complete denture complains falling of upper denture
.Specially during smiling & chatting with his friends and family.What will be the most common
problem associated with complete denture?

a.Too short buccal flange posterior to buccal frenum.

b. Too thick buccal flange posterior to buccal notch area.

c.Overextended PPS

d.Hyperactive buccal frenum musculature

Ans: b. Too thick buccal flange posterior to buccal notch area.

Refer:Zarb&Bolender 12th e ,page no 217

Too thick buccal flange posterior to buccal notch area causes falling of denture.

79.In the following picture two metallic objects in the mandibular anterior ridge represent?

a.Implant abutment

b.Primary healing cap

c.Secondary healing cap

d.These are bullet in to bone.

Ans: b.Primary healing cap

Refer:Misch 3rd e /p 676


Primary Healing cap is given in the above picture.

80.Inflammation around overdenture abutments may be due to all of the following reasons
except?

a.Space around the gingival margin of abutment and denture restoration

b.Excessive or wrong loading of abutments

c.Impingement on the gingival tissue around abutment due to occlusal forces

d.Poor maintaince of oral hygiene

Ans: a.Space around the gingival margin of abutment and denture restoration

Refer: Textbook of Complete Dentures - Page 270, Arthur O. Rahn, John R. Ivanhoe,
Kevin D. Plummer,6th e

Inflammation around overdenture abutments may be due to:

Excessive or wrong loading of abutments


Impingement on the gingival tissue around abutment due to occlusal forces
Poor maintaince of oral hygiene

81.A patient of partially edentulous in posterior mandibular area but maxillary teeth in
opposing arch are so severly extruded that barely 2mm space available for mandibular
teeth.What is your choice of restoration ?

a.Implant retained prosthesis

b.Fixed Dental prosthesis

c.Acrylic partial denture

d.Cast partial denture

Ans: d.Cast partial denture

Refer:McCraken 11th e ,page no 218

Cast partial denture is partially edentulous in posterior mandibular area but maxillary teeth
in opposing arch are so severly extruded that barely 2mm space available for mandibular
teeth.if the no space is available or minimum space is present as in above case ,selected
abutment teeth can be modified to accommodate appropriate clasp assemblies,or
intracoronal retainers can be used.

82.Which of the following is not an advantage of acrylic denture base in cast partial denture?

a.Denture base cannot relined


b.Denture base cannot repaired

c.Better bonding with acrylic teeth

d.Protection of underlying tissue from temperature change

Ans: d.Protection of underlying tissue from temperature change

Refer: Textbook of Prosthodontics - Page 378, Nallaswamy,1st e

The acrylic denture base prostheses have their own advantages and disadvantages. Some
problems with these prostheses are difficult to address, such as insertion in undercut areas,
brittleness of methyl methacrylate which leads to fracture, and allergy to methyl methacrylate
monomer.

Advantage of acrylic denture base in cast partial denture:

a.Denture base cannot relined

b.Denture base cannot repaired

c.Better bonding with acrylic teeth

83.Which type of teeth are used in cast partial denture in severly reduced posterior inter arch
space?

a.Tube teeth

b.Metal pontic

c.Acrylic/plastic teeth

d.Poreclain teeth

Ans: b.Metal pontic

Refer: Planning and Making Crowns and Bridges - Page 215, Bernard G. N. Smith,
Leslie C. Howe,1st e

Metal pontic are used in cast partial denture in severly reduced posterior inter arch space.
The pontic may be fabricated from casting metal or combination of metal and porcelain or
acrylic facing. Usually the full metal pontic is used for the posterior region while the
combination of metal and facing (porcelain or acrylic) is used in anterior region for esthetic
reason. The glazed porcelain is more preferable than acrylic in pontic fabrication because
acrylic is porous in nature and difficult in obtaining highly polished surface which leads to
plaque accumulation and cause gingival Inflammation

84.Stress breaker is not indicated in which kennedys classification:


1) class 1
2) class 2
3) class 4
4) class 6

Ans: Textbook of Prosthodontics - Page 285,Nallaswamy ,1st e

Class VI is an edentulous situation in which the teeth adjacent to the space are capable of
total support of the required prosthesis; it would occur most frequently in a young adult for
whom a fixed partial denture is indicated but possible damage to the dental pulp might occur
if crown preparation were attempted. Most of other RPD are tooth tissue supported
including Kennedy class IV if having long edentulous area crossing midline and need a
stress breaker to equalize the force created on it.

85.In case of xerostomia which of the final impression material is not used?
a) Alginate
b) Zinc oxide eugenol
c) Agar
d) Polyether

Ans: b) Zinc oxide eugenol


Refer: Textbook of Prosthodontics - Page 60.1st e

In case of xerostomia final impression material is not used is Zinc oxide eugenol. Zinc
oxide eugenol is also used as an impression material during construction of complete
dentures and is used in the mucostatic technique of taking impressions, usually in a special
tray, (acrylic) produced after primary alginate impressions. However, ZOE is not usually used
if the patient has large undercuts or tuberosities, whereby silicone impression materials
would be better suited.

86. A patient with complete denture complaints that when he move tongue or lift the tongue,it
leads to lifting of denture.This is most probably due to action on which muscle ?
a.Mylohyoid
b.Superior constrictor
c.Palatoglossus
d.Buccinator

Ans: c.Palatoglossus
Refer:Fenn 2nd e/pg 112

A patient with complete denture complaints that when he move tongue or lift the tongue,it
leads to lifting of denture.This is most probably due to action on which Palatoglossus
muscle.

87. Immediate displacement of lower denture when patient is asked to protrude his tongue
due to?
a.Overextended anterior lingual flange
b.Cramping of tongue
c.Overextended distolingual flange
d.Poor peripheral in retromolar area
Ans: b.Cramping of tongue

Refer: Complete Dentures - Page 90, Hugh Devlin,1st e

Cramped tongue may be due to:

1. Posterior teeth set lingually to the neutral zone.


2. Posterior teeth tilted lingually
3. Posterior teeth too broad bucco-lingual.

88.A patient was diagnosed with hypocalcifiacation type of amelogenesis imperfect comes to
your dental clinic for esthetic correction of her teeth your treatment of choice is?

a.Start calcium orally before any treatment

b.All ceramic crowns in all anterior and posterior teeth.

c.Ceramic laminates in anterior and all ceramic crowns in posterior teeth

d.All ceramic crowns in all anterior teeth only and posterior soft splint to protect the tooth.

Ans: b.All ceramic crowns in all anterior and posterior teeth.

Refer: International Journal of Orthodontics - Volume 12 - Page 25

The best treatment of present condition can be corrected with fixed treatment only additional
calcium supplement or occlusal splint cannot help in patient condition.Best approach must
be anterior all ceramic crowns and posterior metal crowns with ceramic facing.

89.What is wrong about superior constrictor?


a.It is a part of primary muscle of pharynx which help in swallowing.
b.We ask to the patient to push the handle of tray with tongue to record this muscle.
c.its has an attachment with buccinators ,separated by pterygomandibular raphe.
d.Contraction in medial pterygoid leads to create forward bulge in it.
Ans: b.We ask to the patient to push the handle of tray with tongue to record this muscle.

Refer: Textbook of Prosthodontics - Page 58, Nallaswamy,1st e

The superior pharyngeal constrictor muscle is a muscle in the pharynx. It is the highest
located muscle of the three pharyngeal constrictors. The muscle is a quadrilateral muscle,
thinner and paler than the inferior pharyngeal constrictor muscle and middle pharyngeal
constrictor muscle. It is a part of primary muscle of pharynx which help in swallowing.
The superior pharyngeal constrictor muscle is one of the pharyngeal constrictor muscles.
Its primary action is constriction of the pharynx (in coordination with the middle pharyngeal
constrictor and the inferior pharyngeal constrictor muscles) to deliver a bolus of food into the
oesophagus.

The muscle is divided into four parts: A pterygopharyngeal, buccopharyngeal,


myolopharyngeal and a glossopharyngeal part.

The four parts of this muscle arise from:


- the lower third of the posterior margin of the medial pterygoid plate and its hamulus
(Pterygopharyngeal part)
- from the pterygomandibular raphe (Buccopharyngeal part)
- from the alveolar process of the mandible above the posterior end of the mylohyoid line
(Myolopharyngeal part)
- and by a few fibers from the side of the tongue (Glossopharyngeal part)

90.Which of the following cannot be used as denture base material?

a.Poly methy (soft liner)

b.Base metal alloy

c.Poly methyl meth

d..Flexible resin

Ans: a.Poly methy (soft liner)

Refer: Textbook of Complete Dentures - Page 8, Arthur O. Rahn, John R. Ivanhoe,


Kevin D. Plummer/6th e

Poly methy (soft liner) cannot be used as denture base material among all options.

91.During border moulding of lower impression tray handle to activate the?

a.Medial pterygoid

b.superior constrictor

c.palatoglossal

d.Mylohyoid

Ans: d.Mylohyoid

Refer. Boucher's Prosthodontic Treatment for Edentulous Patients,page no.136,1st e


Irrespective of the method used the following border molding movements are carried out
by the dentist:

1) Labial flange is molded by lifting the lower lip outward, upward and inward.

2) The region of the buccal frenum, the cheek is lifted outward, upward, inward, backward
and forward to stimulate movement of frenum.

3) Posteriorly the cheeks are pulled buccaly to ensure that the cheek is not trapped under
the tray and then the cheek is moved upward and inwards.

4) Anterior lingual flange is molded by asking the patient to protrude the tongue and to push
the front part of the palate.

5) Protruding the tongue determines the length of the lingual flange of the tray in this region.

6) Protruding the tongue activates the mylohyoid muscle, which raises the floor of the mouth.
This helps the dentist to determine the length and scope of lingual flange in molar region.

7) If the border molding builds up on the inside of the tray, it must be removed otherwise it
may interfere with the action of mylohyoid muscle.

8) Distal end of lingual flange is molded by asking the patient to protrude the tongue. This
action activated superior constictor muscle which supports the retromylohyoid curtain.

9) The patient is then asked to close as dentist applies downward force on the impression
tray. This records the contraction of medial pterygoid muscle on retromolar curtain.

10) Finally the patient is asked to open wide. If the tray is too long a notch is formed at the
posteromedial border of retromolar pad, indicating encroachment of tray on
pterygomandibular raphe and is adjusted accordingly.

11) Final border molded tray should be so formed that it supports the cheeks and lip in the
same manner as the finished denture will do.

12) The lingual surface of the tray should be shaped so that it guided the tongue in the same
position it will occupy in the final denture.

92. Which of the following is a function of indirect retainer?

a.Prevent movement of denture base away from basal seat when imaginary fulcrum line
established by occlusal rests.

b.Prevent movement away from basal seat when imaginary fulcrum line passes through last
distal abutment

c. Prevent movement away from basal seat by rotation around the vertical axis.

d. Prevent movement away from basal seat by rotation around the horizontal plane axis.
Ans: a.Prevent movement of denture base away from basal seat when imaginary fulcrum
line established by occlusal rests.

Refer: McCracken's Removable Partial Prosthodontics - Page 97/1st e

When the distal extension denture base is dislodged from its basal seat, it tends to rotate
around the fulcrum lines. Theoretically, this movement away from the tissues can be resisted
by activation of the direct retainer, the stabilizing components of the clasp assembly, and the
rigid components of the partial denture framework, which are located on definite rests on the
opposite side of the fulcrum line away from the distal extension base. These components are
referred to as indirect retainers (Indirect retainer components should be placed as far as
possible from the distal extension base, which provides the best leverage advantage against
dislodgment

93.A patient with recent complete denture,has clicking sound whenever teeth come together
as well as pain in TMJ area .What will be the most common error?

a.Excessively large basal area coverge

b.Increase interocclusal space

c.Decrease interocclusal space

d.Overextension of denture flanges

Ans: c.Decrease interocclusal space

Refer:Nallaswamy ,1st e ,page no 130

Vertical dimension of occlusion, or VDO, also known as occlusal vertical dimension


(OVD) is a term used in dentistry to indicate the superior-inferior relationship of the maxilla
and the mandible when the teeth are situated in maximum intercuspation.
A VDO is not only possessed by people who have teeth, however; for completely edentulous
individuals who do not have any teeth with which to position themselves in maximum
intercuspation, VDO can be measured based on subjective signs related to esthetics and
phonetics.

INCREASE IN VDO / DECREASE IN IOD The chin-nose distance will increase, and
thenpatients will have an appearance of openmouth. Constant pressure to the basal seat
area whichwill lead to bone resorption. Soreness of the tissues of the basal seat. Clicking,
of dentures during speech. Improper phonetics

DECREASE IN VDO / INCREASE IN IOD Potentially damaging to the TMJ. The normal
tongue space is limited. Facialdistortion appears more noticeable with overclosure that with
the slightly opened closurebecause with over closure the chin appears to becloser to the
nose, the commissure of the lips turnsdown and the lips lose their fullness. The muscles of
facial expression lose their tonicityand the face appears flabby instead of firm and full. Over
closure of jaws may lead to angular chelitis

94.All are the advantages of using addition silicon as permanent soft liner material except?

a.They does not promote growth of micro-organism like candida albicans

b.Having good bonding properties with acrylic base

c.They are easy to clean with cleaners

d.They are highly resilient materials

Ans: b.Having good bonding properties with acrylic base

Refer: Phillips' Science of Dental Materials - Page 494/11th e

Long-term soft denture lining (LTSDL) materials are used to alleviate the trauma associated
with wearing complete dentures. Despite their established clinical efficacy, the use of
LTSDLs has been limited due to the unfavorable effects of the oral environment on some of
their mechanical and performance characteristics. The unresolved issue of LTSDL
colonization by Candida albicans is particularly problematic. Silicone-based LTSDL
(SLTSDL) materials, which are characterized by more stable hardness, sorption and
solubility than acrylic-based LTSDLs (ALTSDLs), are currently the most commonly used
LTSDLs. However, SLTSDLs are more prone to debonding from the denture base. Moreover,
due to their limitations, the available methods for determining bond strength do not fully
reflect the actual stability of these materials under clinical conditions. SLTSDL materials
exhibit favorable viscoelastic properties compared with ALTSDLs. Furthermore, all of the
lining materials exhibit an aging solution-specific tendency toward discoloration, and the
available cleansers are not fully effective and can alter the mechanical properties of LTSDLs.

Long-term soft denture lining (LTSDL) materials constitute a group of polymer materials that
can remain in the oral cavity for at least four weeks; in practice, however, their use can
extend to several months or even years. The use of LTSDLs is mostly recommended in
edentulous patients with sharp or atrophied alveolar ridges, in patients with thin atrophic
mucosa, in patients in whom the mucosa presents insufficient tolerance to the load
transmitted by the dentures or who experience pain at nerve ending locations, in cases of
the formation of recurrent sore spots under the dentures, in cases in which the denture
exhibits poor retention, as well as for relining in implantology and to perform postoperative
obturation . The application of a soft material is intended to increase the comfort of denture
wearers and to support prosthetic treatment. LTSDLs can help to evenly distribute the biting
loads transferred onto the soft tissues during chewing and to relieve the mucosa from high
mechanical stress. Note that LTSDLs cannot, as has sometimes been suggested, reduce the
forces transmitted by the denture-bearing area, as clearly explained by Braden et al.].
Commercially available dental materials include silicone-based long-term soft denture linings
(SLTSDLs), acrylic-based long-term soft denture linings (ALTSDLs) and, sporadically,
materials based on other polymers. Currently, polymethacrylate materials are used less
frequently and are available as two-component powder-liquid systems. SLTSDLs are
available as one-component materials that cross-link at high temperatures and as two-paste
A-type silicone systems that cross-link at room temperature . Light-cured urethane acrylate
and a polyphosphazene elastomer compounded with methacrylate monomers are other
examples of materials used as LTSDLs

OPERATIVE AND ENDODONTICS

95.If apical 3mm root is resected, then it leads to loss of how many percentage of accessory
canals?

a.86%

b.93%

c.96%

d.98%

Ans: b.93%

Refer:Cohen , 9th e/page no 226

Resection of the apical 3mm of the root apex will eliminate 98% of the apical ramifications
and 93% of the lateral canals which could contain that would contribute to the perradicular
disease.

96.ISO 2 instruments are those who have

a.Taper other than 0.02

b.Contant taper of 0.02 throughout its length

c.Taper continuous variable from starting to end

d.Instruments has two variable taper along its length

Ans: b.Contant taper of 0.02 throughout its length

Refer:Cohen , 9th e , page no 330


ISO 2 instruments are those who have Contant taper of 0.02 throughout its length.An ISO
file was fluted and tapered at 2% for 16 mm.

97.A traumatized tooth which is currently sensitive to electric pulp test mostly become
insensitive to these tests how much time in ?

a.2 weeks

b.3 months

c.6 months

d.2 months

Ans: b.3 months

Refer:Cohen ,10th /page 621

3 months is needed for a traumatized tooth to become insensitive to electric pulp test.These
test should be repeated at 3 weeks, at 3,6 and 12 months and at yearly intervals following
the trauma.

The assessment of pulp health based on its qualitative sensory response is commonly done:

(i)prior to restorative, endodontic, and orthodontic procedures,

(ii)as a follow-up and for monitoring the pulp after trauma to the teeth,

(iii)in differential diagnoses, such as excluding periapical pathosis of pulp origin.

The most accurate way of evaluating the pulp status is by examination of histological
sections of the tissue specimen involved to assess the extent of inflammation or the
presence of necrosis as a means of gauging pulp health. Unfortunately in the clinical
scenario, these are both impractical and not feasible; hence clinicians must use
investigations such as pulp tests to provide additional diagnostic information.

98.In a tooth fracture at mid root level the coronal segment become necrosed and
apical,segment remains intact.What should be most appropriate treatment in this patient?

a.Endodontric treatment of both segment and splint with intra apical root

b.Endo treatment of coronal segment and retrograde MTA filling

c.Endo treatment of coronal sement orthograde MTA filling and removal of apical segment

d.Endo treatment of both parts with MTA

Ans: b.Endo treatment of coronal segment and retrograde MTA filling


Refer:Ingles , 6th e /page no.1341

Endo treatment of coronal segment and retrograde MTA filling should be done in a tooth
fracture at mid root level the coronal segment become necrosed and apical,segment
remains intact.

If MTA is used, the root filling will be placed before healing is seen ,making follow up visits
essential in these cases.In rare cases, when both the coronal and apical pulp areas are
necrotic ,treatment is more complicated.

99.Apicoectomy in endodontics when done with microscopic surgery has magnification of ?

a.8x4

b.8x14

c.14x26

d.26x30

Ans: b.8x14

Refer: Ingle's Endodontics 6 - Page 1289

Apicoectomy in endodontics when done with microscopic surgery has magnification of 8x14

100.Reciprocating rotating technique is used for ?

a.Triangular blank with end cutting tip

b.Triangular blank with non end cutting tip

c.Square blank with end cutting tip

d.Square blank with non end cutting tip

Ans: b.Triangular blank with non end cutting tip

Refer: Cohen's Pathways of the Pulp Expert Consult - Page 232,cohen 10th e

Reciprocating rotating technique is used for Triangular blank with non end cutting tip

101.At what which pulpal pressure does irreversible pulpitis develop ?

a.10 mm Hg

b.15 mm Hg
c.30 mm Hg

d.40 mm Hg

Ans: c.30 mm Hg

Refer:Cohen 9th e/page no 616

Pulpal pressure more than 30 mm Hg for more than 8 hours can cause irreversible pulpitis.

102.A traumatic tooth which is currently sensitive to electric pulp test mostly become
insensitive to these test how much time in?
a.3 months
b.6 months
c.2 months
d.4 months
Ans:a.3 months

Refer: Ingle's Endodontics 6 - Page 537

A traumatic tooth which is currently sensitive to electric pulp test mostly become insensitive
to these test in 3 months.

The primary issues in pulp-vitality testing as follows:

A non-vital post-traumatized incisor has a better long-term prognosis if root canal therapy is
completed before the necrotic pulp gets infected.

The best outcome for the post-traumatizedimmature incisor is for it to revascularize and
continue normal root development, including increased root wall thickness, which is not
possible to assess with conventional electrical and thermal testing20.

Watching and hoping for revascularization using sensitivity testing may lead to infection in
the post-trauma observation period.

PERIODONTICS

103.According to Merlin classification ,class A recall schedule of maintaince visit should be


after?

a.1-3month

b.3-6month

c.6-12month

d.1-2year

Ans: c.6-12month

Refer:Carranza 11th e/page no. 641

Merlin classification :
Class A :Excellent result with well maintained for 1 year

For class B : 3-4 month

Class C: 1-3 month

104.Etiology of gingival response due to increased progesterone and estrogen are all

except?

a.Decrease breakdown of folate

b.Presence of bacteriods melanogenicus

c.Presence of porphyomonas gingivalis

d.Decreased keratinisation

Ans: a.Decrease breakdown of folate

Refer: Carranza's Clinical Periodontology: Expert Consult: Online,12 th/ e ,page no


857

The main sex hormones exerting influence on the periodontium are estrogen and
progesterone. Estrogen and progesterone can significantly influence different organ systems
. For example, estrogens can influence the cytodifferentiation of statified squamous
epithelium, and the synthesis and maintenance of fibrous collagen . Additionally, estrogen
receptors in osteoblast-like cells provide a mechanism for direct action on bone while
estrogen receptors in periosteal fibroblasts and periodontal ligament fibroblasts provide a
mechanism for direct action on different periodontal tissues. Estrogen, progesterone and
chorionic gonadotropin, during pregnancy, affect the microcircularity system by producing
the following changes: swelling of endothelial cells and periocytes of the venules, adherence
of granulocytes and platelets to vessel walls, formation of microthrombi, disruption of the
perivascular mast cells, increased vascular permeability and vascular proliferation .
Consequently, systemic endocrine imbalances may have an important impact on periodontal
pathogenesis, and, vise versa, changes in periodontal conditions might be associated with
variations in sex hormone levels. This association is evident in the recent periodontal
disease classification which includes the following hormone related disease categories:
puberty-associated gingivitis, menstrual cycle-associated gingivitis and pregnancy-
associated gingivitis. Puberty gingivitis is characterized clinically by the onset of exuberant
inflammation of the marginal and, by direct extension, adjacent attached gingiva, especially
in the interdental papillae , with increased gingival bleeding during puberty . This gingival
enlargement, is found primarily on the facial surfaces, with the lingual surfaces remaining
relatively unaltered

105.Which of the following is not an example of universal curette?

a.Columbia #1R-2L
b.Young wood #7-8

c.Indiana university #17-18

d.Mc call#13 -14

Ans: b.Young wood #7-8

Refer:Carranza's Clinical Periodontology: Expert Consult: Online


By Michael G. Newman, Henry Takei, Perry R. Klokkevold, Fermin A. Carranza,page no
497 /11th e

Universal Curettes:

Universal curettes have cutting edges that may be inserted in most areas of the dentition by
altering and adapting the finger rest, fulcrum, and hand position of the operator. The blade
size and the angle and length of the shank may vary, but the face of the blade of every
universal curette is at a 90-degree angle (perpendicular) to the lower shank when seen in
cross-section from the tip. The blade of the universal curette is curved in one direction from
the head of the blade to the toe. The Barnhart curettes #1-2 and 5-6 and the Columbia
curettes #13-14, 2R-2L, and 4R-4L are examples of universal curettes. Other popular
universal curettes are the Younger-Good #7-8, McCalls #17-18, and the Indiana University
#17-18

106.Which of the following interleukin family is related to inflammation?

a.IL1-f4

b.IL1-f6

c.IL-f8

d.IL-f10

Ans: a.IL1-f4

Refer: Carranza's Clinical Periodontology,page no 346/11th e

IL1-f4 interleukin family is related to inflammation.

107.Dimension of periochip is?

a.3 4 0.5mm

b. 4 5 0.25mm

c. 3 4 1mm

d. 5 4 0.35mm

Ans: d. 5 4 0.35mm
Refer: Periodontics: Medicine, Surgery, and Implants - Page 282,1st e , Louis F. Rose

The chip measures 4 mm in length, 5 mm in width and 0.250.35 mm in thickness (4 mm 5


mm 0.250.35 mm), and a single chip weighs about 10 mg.

108.Which of the following inflammatory cells are predominantly involved in chronic


periodontitis?

a.Neutrophils

b.Plasma cells

c.Eosinophils

d.Lymphocytes

Ans: b.Plasma cells

Refer: Essentials of Oral Pathology - Page 532, By Swapan Kumar Purkait,3rd e

Plasma cells inflammatory cells are predominantly involved in chronic periodontitis. The
inflammatory cell infiltrate, which is predominantly plasma cells, extends deeper into the
connective tissue. This results in widespread manifestations of inflammation and
immunopathological tissue damage.

109.Which of the following is not a risk indicator for periodontits?

a.Infrequent dental visits

b.Bleeding on probing

c.HIV

d.Osteoporosis

Ans: a.Infrequent dental visits

Refer: Hall's Critical Decisions in Periodontology - Page 106,5th e

Risk indicators :Tobacco smoking, Genetic factors HIV/AIDS, Previous history of periodontal
disease ,Diabetes, Age, Osteoporosis ,Bleeding on probing Pathogenic bacteria, Gender
Infrequent dental visits Microbial tooth deposit ,Socioeconomic status, Stress.

Tooth loss due to periodontal disease is associated with the risk indicators of age, male
gender, smoking, lack of professional maintenance, inadequate oral hygiene, diabetes
mellitus, hypertension, rheumatoid arthritis and anterior tooth type.

110.Dental plaque contains?

a.Bacteria and food products


b.Bacteria and bacterial products

c.Pellicle and food

d.Salivary products and food particles

Ans: b.Bacteria and bacterial products

Refer: Carranza 11th ed 316 / 10th ed 137

Dental plaque contains Bacteria and bacterial products. Dental plaque is the material that
adheres to the teeth and consists of bacterial cells (mainly S. mutans and S. sanguis),
salivary polymers and bacterial extracellular products. Plaque is a biofilm on the surfaces of
the teeth. This accumulation of microorganisms subject the teeth and gingival tissues to high
concentrations of bacterial metabolites which results in dental disease. If not taken care of,
via brushing or flossing, the plaque can turn into tartar (its hardened form) and lead to
gingivitis or periodontal disease.

111.Adult onset tooth discoloration is seen in?

a.Long use of minocycline for treatment of acne.

b.Long-Time use of doxycycline for treatment of chronic periodontitis

c.Use of tetracycline during entire pregnancy.

d.Due to formation of by-products of tetracycline use during childhood.

Ans: a.Long use of minocycline for treatment of acne.

Refer: Endodontics: Principles and Practice - Page 393/4th e , Mahmoud Torabinejad,


Richard E. Walton

The remarkable side-effect of minocycline on the oral cavity is the singular occurrence of
"black bones", "black or green roots" and blue-gray to gray hue darkening of the crowns of
permanent teeth. The prevalence of tetracycline and minocycline staining is 3-6%. The
mechanism of minocycline staining is still unknown.

112.All of the following can be included for active tissue engineering purpose in periodontal
procedure except?

a.BMP

b.Autologus fibroblast

c.Enamel matrix derivative

d.Biological dermal acellular graft

Ans: d.Biological dermal acellular graft


Refer: Contemporary Implant Dentistry - Page 868, Carl E. Misch,3rd e

All of the following can be included for active tissue engineering purpose in periodontal
procedure:

a.BMP

b.Autologus fibroblast

c.Enamel matrix derivative

Several bioactive molecules have demonstrated strong effects in promoting periodontal


wound repair in preclinical and clinical studies. These bioactive molecules include PDGF ,
IGF-I , basic fibroblast growth factor (FGF-2) , TGF-1 , BMP-2 , -4 , -7 and -12 , and enamel
matrix derivative (EMD) that have shown positive results in stimulating periodontal
regeneration. In addition, PDGF, BMP-2, and BMP-7 have been shown to promote peri-
implant bone regeneration . Recently, a comparative study in periodontal defects found that
rhBMP-2 in association with collagen sponges promoted superior bone regeneration than
rhBMP-12. Cementum formation is similar for both BMPs, however, only the rhBMP-12
treatment induces to functionally oriented PDL bridging newly formed bone and new
cementum

113.Which of the following are not involved in early colonization of plaque?

a.Strep mitis

b.strep intermedius

c.strep Gordonii

d.Strep. conestellatus

Ans: d.Strep. conestellatus

Refer: Carranza's Clinical Periodontology,page no.394/11th e

Strep mitis,strep intermedius,strep Gordonii involved in early colonization of plaque.Healthy


tooth surfaces and gingivae tend to only be associated with this first phase of biofilm
development. It consists of an initial few layers (1-20) of mostly gram-positive cocci bacteria,
followed by some gram-positive rods and fillaments and a very small amount of gram-
negative cocci. The gram-positive cocci species involved in this conditioning layer include,
but are not limited to, Streptococcus mutans, Streptococcus mitis, Streptococcus sanguis,
Streptococcus oralis, Rothia dentocariosa, and Staphylococcus epidermidis. The gram-
negative rod and filament species include Actinomyces viscosus, Actinomyces israelis.
Actinomyces gerencseriae and the Corynebacterium species. Veillonella parvula and the
Neisseria species make up some of the few gram-negative cocci, which are aerobes or
facultative aerobes and are able to adhere to the non-exfoliating hard tooth surfaces [5]. This
early composition of the biofilm is able to withstand many of the frequent mechanisms of the
oral cavity that contribute to bacterial removal such as swallowing, nose blowing, chewing,
and salivary fluid outflow. The early colonizers are also able to survive in the high oxygen
concentrations present in the oral cavity, without having much protection from other bacteria
[5]. Thus, this thin, initial biofilm is almost always present on the tooth surface as it forms
immediately after cleaning.

114.Most predominant microflora in juvenile periodontitis?

a.Aerobic gram negative cocci

b. Anaerobic gram positive cocci

c. Aerobic gram positive cocci

d. Anaerobic gram negative cocci

Ans: d. Anaerobic gram negative cocci

Refer: Carranza 11th ed 239 / 10th ed 502

Most predominant microflora in juvenile periodontitis anaerobic gram negative cocci.

115.Bacterial level is reduced from blood ,after how much time of periodontal/dental
intervention?

a.10min

b.20min

c.30min

d.60min

Ans: c.30min

Refer: Carranza's Clinical Periodontology,11th e ,page no .835

Bacterial level is reduced from blood ,after 30 minutes of periodontal/dental intervention.

116.What will be the radiographic feature for chronic gingivitis ?

a.Horizontal bone loss


b.Vertical bone loss

c.normal bone pattern

d.wedge shape bone loss

Ans: c.normal bone pattern

Refer: Carranza's Clinical Periodontology,11th e ,page no .629

Bone loss is seen in periodontitis not in gingivitis.

117. Gene responsible for negative regulation of BMP1 during demineralisation by:
A. Noggins
B. Amelogenin
C. EMD
D. Cbfa 1
Ans: A. Noggins
Refer:Lindhe 5th e /page no. 558

Bone morphogenetic protein 4 is a protein that in humans is encoded by BMP4 gene.


BMP4 is found on chromosome 14q22-q23

BMP4 is a member of the bone morphogenetic protein family which is part of the
transforming growth factor-beta superfamily. The superfamily includes large families of
growth and differentiation factors. BMP4 is highly conserved evolutionarily. BMP4 is found in
early embryonic development in the ventral marginal zone and in the eye, heart blood and
otic vesicle

Bone morphogenetic protein 4 is a protein that in humans is encoded by BMP4 gene.


BMP4 is found on chromosome 14q22-q23

BMP4 is a member of the bone morphogenetic protein family which is part of the
transforming growth factor-beta superfamily. The superfamily includes large families of
growth and differentiation factors. BMP4 is highly conserved evolutionarily. BMP4 is found in
early embryonic development in the ventral marginal zone and in the eye, heart blood and
otic vesicle. BMPs acting through an autocrine mechanism reduce the expression of
collagenase-3 and noggin, thereby inhibiting BMP binding and function and, result in
increased production of collagenase-3 . Although BMPs exert their action on both
osteoblasts and chondroblasts, they do not change the fate of the respective progenitors .
Early exposure of undifferentiated mesenchymal cells to BMPs induces the chondroblastic
pathway, whereas later exposure accelerates osteoblastic differentiation . BMPs can
stimulate osteoblast differentiation independently of cartilage formation.

118.Which is the least critical zone in pocket elimination procedure?


a) Tooth Surface
b) Root surface
c) Underlying Bone
d) Pocket tissue wall
Ans: d) Pocket tissue wall
Refer:Carranza ,10th e ,page no 884
Pocket tissue wall is the least critical zone in pocket elimination procedure.
Criteria for the selection of one of the different surgical techniques for pocket therapy are
based on clinical findings in the soli tissue pocket wall, tooth surface, underlying bone, and
attached gingiva.

Zone 1: The Soft Tissue Wall of the morphologic features, thickness, and topography of the
soft tissue pocket wall and persistence of inflammatory changes in it should be determined.

Zone 2: The Tooth Surface of the presence of deposits and alterations on the cemen-turn
surface and the accessibility of the root surface to instrumentation should be identified.
Phase I therapy should have solved many, il not all. of the problems on the tooth surface. I
valuation of the results of Phase I therapy should determine the need for further therapy and
the method to be used.

Zone 3: The Bone

The shape and height of the alveolar bone next to the pocket wall should be established by
careful probing and clinicoradiographic examination, craters, horizontal or angular bone
losses, and other bone deformities are important criteria lor the selection of the treatment
technique.

Zone 4: The Attached Gingiva

The presence or absence of an adequate band of attached gingiva is a factor to be


considered when selecting the pocket treatment method. Diagnostic techniques for
mucogingival problems are described in (hapter 6ft. An inadequate attached gingiva may he
due to a high frenum attachment, marked gingival recession, or a deep pocket that reaches
the level of the mucogingival junction. All of these possible conditions should be explored
and their influence on pocket therapy determined.

119.Not a risk indicator for periodontitis


a) Infrequent dental visits
b) Bleeding on probing
c) HIV
d) Osteoporosis
Ans:a) Infrequent dental visits

Refer: Carranza's Clinical Periodontology: Expert Consult: Online


By Michael G. Newman, Henry Takei, Perry R. Klokkevold, Fermin A. Carranza,page no
391, 12th e

A risk factor can be defined as an occurrence or characteristic that has been associated with
the increased rate of a subsequently occurring disease. It is important to make the distinction
that risk factors are associated with a disease but do not necessarily cause the disease. Risk
factors may be modifiable or non-modifiable. Modifiable risk factors are usually
environmental or behavioral in nature whereas non-modifiable risk factors are usually
intrinsic to the individual and therefore not easily changed. Non-modifiable risk factors are
also known as determinants. Smoking is the best established of the modifiable risk factors
for developing periodontal disease. In addition to smoking, evidence exists which suggests a
relationship between periodontal status and nutrition, alcohol consumption, socioeconomic
status and stress levels, although these relationships have not been clearly established.
Bleeding on probing,HIV and Osteoporosis are also included in risk factors for periodontitis.

ORAL PATHOLOGY

120.Most commonly encountered histological variant of ameloblastoma is?

a.Plexiform

b.granulomatous

c.acanthomatous

d. follicular

Ans: d. Follicular

Refer: Shafer's Textbook of Oral Pathology,6th e - Page 280

Most commonly encountered histological variant of ameloblastoma is Follicular.

Least commonly encountered histological variant of ameloblastoma is Basal.

121.Histological variant of ameloblastoma which has high recurrence rate?

a.Follicular

b.Plexiform

c.Acanthomatous

d.Granular cell

Ans: d.Granular cell

Refer: Shafer's Textbook of Oral Pathology,6th e - Page 280

Histological variant of ameloblastoma which has high recurrence rate Granular cell.

122. This cyst is now considered as the posterior extension of incisive canal cyst
a. Median palatal cyst
b. Nasolabial cyst
c. Globulomaxillary cyst
d. Median alveolar cyst
Ans: a. Median palatal cyst

Refer: Textbook of Oral Medicine - Page 210, Ghom,3rd e

Median palatine cysts are rare, nonodontogenic lesion of the hard palate that do not involve
the palatine papilla or incisive canal and that usually present as asymptomatic, fluctuant
swellings. They may involve the floor the nasal cavity but are treated by simple enucleation,
without recurrence. They are composed histologically of a fibrous collagenous tissue wall,
with infiltration of chronic inflammatory cells, and lined by stratified squamous and/or
respiratory epithelium. They are differentiated from the nasopalatine and other anterior
maxillary cysts by the following diagnostic criteria: Appears grossly to be symmetrical along
the midline of the hard palate. Located posterior to palatine papilla. Radiographically ovoid or
circular in appearance. Not intimately associated with a nonvital tooth or found to have any
communication with the incisive canal. Shows no histologic evidence of nerve trunks, large
vascular spaces, hyaline cartilage, or accessory salivary gland tissue in the cyst wall.

123. Sickle cell anaemia is


a.autosomal recessive gene mutation
b.autosomal recessive point mutation
c.autosomal dominant point mutation
d.autosomal dominant gene mutation
Ans: b.autosomal recessive point mutation

Refer: Shafer'S Textbook Of Oral Pathology (6Th Edition) - Page 762

SCA is an autosomal recessive disease caused by a point mutation in the hemoglobin beta
gene (HBB) found on chromosome 11p15.5. Carrier frequency of HBB varies significantly
around the world, with high rates associated with zones of high malaria incidence, since
carriers are somewhat protected against malaria. About 8% of the African American
population are carriers. A mutation in HBB results in the production of a structurally abnormal
hemoglobin (Hb), called HbS. Hb is an oxygen carrying protein that gives red blood cells
(RBC) their characteristic color. Under certain conditions, like low oxygen levels or high
hemoglobin concentrations, in individuals who are homozygous for HbS, the abnormal HbS
clusters together, distorting the RBCs into sickled shapes. These deformed and rigid RBCs
become trapped within small blood vessels and block them, producing pain and eventually
damaging organs.

124. which of the following marker is present in parakeratined area but absent in
orthokeratinised area in oral mucosa
A K10
B K19
C K12
D K23
Ans: b.K19

Refer: Shafer's Textbook of Oral Pathology - Page 97,6th e

K19 is an intermediate filament protein that has been investigated in oral squamous cell
carcinoma (OSCC), but that has not been correlated with the amount of keratin produced
within well-differentiated OSCC grade. When K19 is produced by suprabasal cells of the oral
mucosa, this indicates alteration in cell behavior and probable premalignant changes. K19 is
an intermediate protein normally present in glandular epithelium and in undifferentiated stem
cells as well as in the basal cell layer of oral mucosa.

125.Differential WBC count is useful in the diagnosis of?


a.Anemia
b.Thrombocytopenia purpura
c.Spherocytosis
d.Eosinophila
Ans: d.Eosinophila
Refer: Shafer 7th ed 283/6th ed 279

Differential WBC count is useful in the diagnosis of Eosinophila Eosinophilia Eosinophilia


refers to an absolute eosinophil count in the peripheral blood of 500 eosinophils/microL;
this is considered abnormal in most laboratories . The degree of eosinophilia can also be
categorized as mild (500 to 1500 eosinophils/microL), moderate (1500 to 5000
eosinophils/microL), or severe (>5000 eosinophils/microL).

The typical percentage of eosinophils in an individual without eosinophilia is less than 5


percent, but the presence of eosinophilia cannot be determined based on the eosinophil
percentage alone, because this percentage is a relative number that varies with the total
WBC count and the relative percentages of other WBCs (eg, neutrophils, lymphocytes).

126.A child has permanent anteriors small in size ,while all primary teeth were normal .This
is which type of hypodontia?
a.Generalized developmental hypoplasia
b.Hypocalcification
c.Focal developmental defects
d.Hypo maturation or hypoplasia

Ans: c.Focal developmental defects


Refer:Neville , 2nd e ,page no 74-75
Gemination is defined as a enlarged tooth or joined tooth in which tooth count is normal
when the anomalous tooth is counted as one.Fusion is defined as a single enlarged tooth or
toned tooth in which the tooth count reveals missing tooth when anomalous tooth is counted
as one .

127.According to latest literature and research ,klesdent cyst is derived from?


a.Maxillary sinus epithelium
b.Remnants of nasolacrimal duct
c.Cell rest of serre
d.Rest of malaseez
Ans: b.Remnants of nasolacrimal duct
Refer:Cysts of the oral and maxillofacial regions Mervyn shear, 4thy e ,page no.121

Klesdent cyst is derived from Remnants of nasolacrimal duct.

128.Eye looking to heaven appearance occurs in ?


a.Pagets disease
b.Burkitt lymphoma
c.Cherubism
d.Fibrous dysplasia

Ans: c.Cherubism

Refer: Oral Radiology: Principles and Interpretation - Page 419, Stuart C. White,
Michael J. Pharoah,6th e

Eye looking to heaven appearance occurs in Cherubism.


Cherubism is an inherited, fibro-osseous condition characterized by firm, painless swelling of
the jaws, and was first described by Jones in 1933. Cherubism is usually diagnosed in the
first decade of life. Clinically cherubism is characterized by fullness of the cheeks and jaw
bones that results in a round face with retraction of the lower eyelids and exposure of the
sclera below the irises; the raised- to- heaven look produced is suggestive of a cherub and
gave rise to the name of this condition. The radiographical appearance is characterized by
bilateral, multilocular radiolucent lesions, which often begin near the angle of the mandible
and spread to the ramus and body of the mandible. Expanding maxillary lesions can result in
obliteration of maxillary sinuses. The presence of eosinophilic, collagenous material around
small capillaries in histopatholgical sections is of value in the diagnosis of cherubism

129.The Most common cyst occurring in head neck region??


a) Brachial
b) Thyroglossal
c) Dermoid
d) Lymphoepithelial Cyst

Ans: b) Thyroglossal

Refer: Shafer's Textbook of Oral Pathology ,6th e - Page 68

Thyroglossal duct cysts (TGDC's) are the most common congenital neck cyst. They are
typically located in the midline and are the most common midline neck mass in young
patients. They can be diagnosed with multiple imaging modalities, including ultrasound, CT,
and MRI.
130.Giant cell tumor and giant cell granuloma can be diffferntiated by all except?

a.Number of nuclei in giant cell.

b.Focal area of giant cell.

c.Tumor necrosis and cell of inflammation

d.Size and shape of giant cells

Ans: c.Tumor necrosis and cell of inflammation

Refer:Neville 2e/page no 544

Common to all presence of few to many multinucleated giant cells in a background of ovoid
to spindle shaped shaped mesenchymal cells.The giant cells may be aggregated focally in
the lesional tissue or may be presently diffusely throughout the lesion.These cells vary
considerably in size and shape from case to case.

Some are small and irregular in shape and contain 20 or more nuclei.

131.In the given picture the horizontal linear destruction of enamel surface (DARK YELLOW
LINE )is mostly due to:

a.Hypophosphatemia

b.Fluorosis

c.Chicken pox

d.Pagets disease

Ans: b.Fluorosis
Refer: Textbook of Preventive and Community Dentistry - Page 216, S. S. Hiremath,2nd
e

Fluorosis is a cosmetic condition that affects the teeth. Its caused by overexposure to
fluoride during the first eight years of life. This is the time when most permanent teeth are
being formed.

After the teeth come in, the teeth of those affected by fluorosis may appear mildly discolored.
For instance, there may be lacy white markings that only dentists can detect. In more severe
cases, however, the teeth may have:

Stains ranging from yellow to dark brown


Surface irregularities

Pits that are highly noticeable

132.Another name of encephalotrigemminalangiomatosis is?

a.Rendu-osler weber syndrome

b.Sturge-weber syndrome

c.Beckwith weidmann syndrome

d.Chikwchik syndrome

Ans: b.Sturge-weber syndrome

Refer: Shafer 7th ed 149/6th ed 144

SturgeWeber syndrome, sometimes referred to as encephalotrigeminal angiomatosis,


is a rare congenital neurological and skin disorder. It is one of the phakomatoses and is often
associated with port-wine stains of the face, glaucoma, seizures, mental retardation, and
ipsilateral leptomeningeal angioma (cerebral malformations and tumors). Sturge Weber
Syndrome can be classified into three different types. Type 1 includes facial and
leptomeningeal angiomas as well as the possibility of glaucoma or choroidal lesions.
Normally, only one side of the brain is affected. This type is the most common. Type 2
involvement includes a facial angioma (port wine stain) with a possibility of glaucoma
developing. There is not any evidence of brain involvement. Symptoms can show at any time
beyond the initial diagnosis of the facial angioma. The symptoms can include glaucoma,
cerebral blood flow abnormalities and headaches. More research is needed on this type of
Sturge Weber Syndrome. Type 3 has leptomeningeal angioma involvement exclusively. The
facial angioma is absent and glaucoma rarely occurs. This type is only diagnosed via brain
scan.

133.Neurotransmitter used in Freys Syndrome is:

a.Acetylcholine

b.Epinephrine
c.Nor epinephrine

d.Serotonin

Ans: a.Acetylcholine

Refer: Shafer 7th ed 863/6th ed 854

Frey syndrome is characterized by erythema, flushing, and gustatory sweating in the


preauricular area. It most commonly occurs after parotid surgery. A variety of medical and
surgical modalities have been used to manage the symptoms of Frey syndrome. Topical
therapies target either the cholinergic synapse via acetylcholine receptor antagonism
(scopolamine, atropine, and glycopyrrolate) or the eccrine gland via direct inhibition of
eccrine gland secretion (aluminum chloride solutions and other antiperspirants). Topical
scopolamine and atropine have the disadvantage of variable efficacy and increased risk of
adverse side effects including xerophthalmia, xerostomia, blurred vision, and urinary
retention.Topical preparations of glycopyrrolate, ranging from 0.5 to 2% in either liquid or
cream, applied once daily, every other day, or even once weekly have been found to be
efficacious with few adverse effects.Topical aluminum chloride, either as a 12 or 20%
solution, has also been beneficial for treating syndrome-associated symptoms with minimal
or no side effects.

134.Ab luminal cells in pleomorphic adenoma will differentiate into?

a.Mucin cells

b.Plasmacytoid cells

c.Sebaceous cells

d.Glandular cells

Ans: Mucin cells

Refer: Shafer 7th ed 229/6th ed 2

The tumor cells that differentiate into myoepithelial cells (neoplastic myoepithelial cells) are
one of the unique pathological features of salivary gland tumors. Neoplastic myoepithelial
cells by themselves do not demonstrate glandular formation, but are located around the
ductal cells in the gland-forming tumors (Fig. 2CH). The cells show various morphologies,
such as epitheloid, spindle, plasmacytoid and clear cell features, and frequently produce a
mucinous or basement membrane-like extracellular matrix. Neoplastic myoepithelial cells
can sometimes be identified by HE staining, but an IHC analysis is often necessary for a
more accurate identification.

135.Which chromosome is involved in dentinogenesis imperfect?

a.Chromosome 4

b.Chromosome 6

c. Chromosome 9
d. Chromosome 21

Ans: a.Chromosome 4

Refer: Shafer 7th ed 58/ 6th ed 57

Dentinogenesis imperfecta type III (DGI-III) is an autosomal-dominant disorder of dentin


formation which appears in a tri-racial southern Maryland population known as the
"Brandywine isolate". This disease has suggestive evidence of linkage to the long arm of
human chromosome 4 (LOD score of 2.0) in a family presenting with both juvenile
periodontitis and DGI-III. The purpose of this study was to screen a family presenting with
only DGI-III to determine if this locus was indeed on chromosome 4q.

136.A patient having history of hypercalcemia ,ground glass appearance of bone


,generalised hypoplasia of teeth is usually suffered from?

a.paget disease

b.Hyperparathyroidism

c.Hypoparathyroidism

d.Sickle cell anemia

Ans: b.Hyperparathyroidism

Refer: Shafer's 7th ed 653/6th ed 652

X-rays may also be used to diagnose the disease. Usually, these X-rays will show extremely
thin bones, which are often bowed or fractured. However, such symptoms are also
associated with other bone diseases, such as osteopenia or osteoporosis. Generally, the first
bones to show symptoms via X-ray are the fingers.Furthermore, brown tumors, especially
when manifested on facial bones, can be misdiagnosed as cancerous.Radiographs distinctly
show bone resorption and X-rays of the skull may depict an image often described as
"ground glass" or "salt and pepper"

137.Hypothyroidism in children causes?

a.Cretinism

b.Myxoedema

c.Acromegaly

d.Giagantism

Ans: a.Cretinism

Refer: Shafer 7th ed 52/6th ed 51


Congenital hypothyroidism (CH) or cretinism is a condition of thyroid hormone deficiency
present at birth. Approximately 1 in 4000 newborn infants has a severe deficiency of thyroid
function, while even more have mild or partial degrees. If untreated for several months after
birth, severe congenital hypothyroidism can lead to growth failure and permanent intellectual
disability. Treatment consists of a daily dose of thyroid hormone (thyroxine) by mouth.
Because the treatment is simple, effective, and inexpensive, nearly all of the developed
world practices newborn screening to detect and treat congenital hypothyroidism in the first
weeks of life.

138.Which of the following has high threshold fibres?

a.A alpha

b.A Delta

c.C Fibres

d.A gamma

Ans: c.C Fibres

Refer: Shafer'S Textbook Of Oral Pathology (6Th Edition) - Page 374

Group C nerve fibers are one of three classes of nerve fiber in the central nervous system
and peripheral nervous system. The C group fibers are unmyelinated and have a small
diameter and low conduction velocity. They include Postganglionic fibers in the autonomic
nervous system (ANS), and nerve fibers at the dorsal roots (IV fiber). These fibers carry
sensory information.

Damage or injury to nerve fibers causes neuropathic pain. Capsaicin activates C fibers
Vanilloid receptors, giving chili peppers a hot sensation.

139.An adolescent patient comes to clinic and complains of orange yellow discoloration of
permanent teeth ,having history of hospitalization for chronic disease when she was 4-5
years of age.it is due to which of the following condition?

a.Pellagra

b.Jaundice

c.Anemia

d.Leukemia

Ans: b.Jaundice

Refer: Shafer'S Textbook Of Oral Pathology (6Th Edition) - Page 560


Congenital hyperbilirubinaemia: The breakdown products of haemolysis will cause a yellow-
green discolouration. Mild neonatal jaundice is relatively common, but in rhesus
incompatibility massive haemolysis will lead to deposition of bile pigments in the calcifying
dental hard tissues, particularly at the neonatal line.

140.20%KOH mount used for rapid evaluation of candida fungus causes?

a.Staining of candida hyphae

b.Destruction of hyphal tissue

c.Dissolution of underlying epithelial tissue

d.Fluorescence

Ans: c.Dissolution of underlying epithelial tissue

Refer: Microbiology and Immunology,page no 138,1st e

The KOH Test for Candida albicans, also known as a potassium hydroxide preparation or
KOH prep, is a quick, inexpensive fungal test to differentiate dermatophytes and Candida
albicans symptoms from other skin disorders like psoriasis and eczema. 20%KOH mount
used for rapid evaluation of candida fungus causes Dissolution of underlying epithelial
tissue.

The KOH test procedure may be performed by a physician, nurse practitioner, physician
assistant, medical assistant, nurse, or medical laboratory technician. If fungal cultures are
required, the test is performed by a technologist who specializes in microbiology.

1. Collection: Skin, nail, or hair samples are collected from the infected area on the
patient. For skin samples, a scalpel or edge of a glass slide is used to gently scrape
skin scales from the infected area. For hair samples, a forceps is used to remove hair
shafts and follicles from the infected site. If the test is being sent to a laboratory, the
scrapings are placed in a sterile covered container.
2. The scrapings are placed directly onto a microscope slide and are covered with 10%
or 20% potassium hydroxide.

3. The slide is left to stand until clear, normally between five and fifteen minutes, in
order to dissolve skin cells, hair, and debris.

4. To enhance clearing dimethyl sulfoxide can be added to the slide. To make the fungi
easier to see lactophenol cotton blue stain can be added.

5. The slide is gently heated to speed up the action of the KOH.

6. Adding calcofluor-white stain to the slide will cause the fungi to become fluorescent,
making them easier to identify under a fluorescent microscope.
7. Place the slide under a microscope to read

141.Spindle cell tumor is?

a.Glomus tumor

b.Small cell shwannoma

c.Melanoma

d.Adenocarcinoma/melanocarcinoma

Ans: c.Melanoma

Refer: Shafer 7th ed 123/6th ed 118

Spindle cell tumor is Melanoma.

Spindle cell melanoma, also known as desmoplastic melanoma, is a rare form of melanoma
that is locally aggressive and tends to have a high recurrence rate even with treatment.

This type of melanoma the name refers to the spindle-like shape of the cancer cells is
most commonly found on sun-exposed skin, in particular the head and neck. Because these
lesions can involve nerves, they tend to be more symptomatic than regular melanomas.
Spindle cell melanomas may arise from a preexisting lesion or without a precursor. These
lesions may be pigmented or flesh-colored. This subtype of melanoma is most common in
elderly, male patients, but women are at risk as well.

All melanoma lesions should be treated aggressively because of their risk of spreading to
other parts of the body. Spindle cell melanomas tend not to spread throughout the body as
readily as other types of melanomas; however, they often recur locally and can be fatal
depending on the location and if the local invasion is deep enough. Unfortunately, these
lesions are usually deep by the time the diagnosis is made.

142.A patient reported to hospital with very few teeth in mouth and inability to sweat. The
patient is affected with?

a.Down syndrome

b.Klinfilter syndrome

c.Ectodermal dyaplasia

d.Treacher Collins syndrome

Ans: c.Ectodermal dyaplasia

Refer: Shafer 7th ed 809/6th ed 800


Ectodermal dysplasia is not a single disorder, but a group of syndromes all deriving from
abnormalities of the ectodermal structures. More than 150 different syndromes have been
identified.

Despite some of the syndromes having different genetic causes the symptoms are
sometimes very similar. Diagnosis is usually by clinical observation often with the assistance
of family medical histories so that it can be determined whether transmission is autosomal
dominant or recessive.

Ectodermal dysplasias are described as "heritable conditions in which there are


abnormalities of two or more ectodermal structures such as the hair, teeth, nails, sweat
glands, cranial-facial structure, digits and other parts of the body.

Presentation

Hair

Individuals affected by an ED syndrome frequently have abnormalities of the hair follicles.


Scalp and body hair may be thin, sparse, and very light in color, even though beard growth in
affected males may be normal. The hair may grow very slowly or sporadically and it may be
excessively fragile, curly, or even twisted.

Nails

Fingernails and toenails may be thick, abnormally shaped, discolored, ridged, slow-growing,
or brittle. The cuticles may be prone to infections.

Skin

The skin may be lightly pigmented. Skin sustaining injury may grow back permanently hypo-
pigmented. In some cases, red or brown pigmentation may be present. Skin can be prone to
rashes or infections and can be thick over the palms and soles. Care must be taken to
prevent cracking, bleeding, and infection.

Sweat glands

Individuals affected by certain ED syndromes cannot perspire. Their sweat glands may
function abnormally or may not have developed at all because of inactive proteins in the
sweat glands. Without normal sweat production, the body cannot regulate temperature
properly. Therefore, overheating is a common problem, especially during hot weather.
Access to cool environments is important.[4]
Dental abnormalities in a 5-year-old girl from north Sweden family who suffered from various
symptoms of autosomal dominant hypohidrotic ectodermal dysplasia (HED) a) Intraoral view.
Note that the upper incisors have been restored with composite material to disguise their
original conical shape. b) Orthopantomogram showing absence of ten primary and eleven
permanent teeth in the jaws of the same individual.

Teeth

In the development of tooth buds frequently result in congenitally absent teeth (in many
cases a lack of a permanent set) and/or in the growth of teeth that are peg-shaped or
pointed. The enamel may also be defective. Cosmetic dental treatment is almost always
necessary and children may need dentures as early as two years of age. Multiple denture
replacements are often needed as the child grows, and dental implants may be an option in
adolescence, once the jaw is fully grown. Nowadays the option of extracting the teeth and
substituting them with dental implants is quite common. In other cases, teeth can be
crowned. Orthodontic treatment also may be necessary. Because dental treatment is
complex, a multi-disciplinary approach is best.

143.A patient with ectodermal dysplasia shows one or two malformed teeth in his mouth.On
radiographic examination patient shows?

a.Mutliple unerupted supernumerary teeth

b.No radiographic evidence of teeth germ of primary or permanent teeth

c.Underdeveloped condyle of mandible

d.Normal dentition with delayed or arrested eruption.

Ans:b.No radiographic evidence of teeth germ of primary or permanent teeth


Refer:Neville 3rd e/page no 7742

A patient with ectodermal dysplasia shows one or two malformed teeth in his mouth.On
radiographic examination patient shows No radiographic evidence of teeth germ of primary
or permanent teeth.

144.Round cell tumor are all except?

a.Leiomyoma

b.Ewing sarcoma

c.Lymphoma

d.PNET

Ans: a.Leiomyoma

Refer: Shafer's Textbook of Oral Pathology - Page 169, By Arya Rajendran, B


Sivapathasundharam,/7th e

Malignant small round cell tumors are characterised by small, round, relatively
undifferentiated cells. They generally include Ewing's sarcoma, peripheral neuroectodermal
tumor, rhabdomyosarcoma, synovial sarcoma, non-Hodgkin's lymphoma, retinoblastoma,
neuroblastoma, hepatoblastoma, and nephroblastoma or Wilms tumor. Other differential
diagnoses of small round cell tumors include small cell osteogenic sarcoma, undifferentiated
hepatoblastoma, granulocytic sarcoma, and intraabdominal desmoplastic small round cell
tumor. Differential diagnosis of small round cell tumors is particularly difficult due to their
undifferentiated or primitive character.

145.Hunters and Driffield curve is related to ?

a.Dental caries

b.Dental radiograph

c.Ni-Ti wire

d.stainless steel

Ans: b.Dental radiograph

Refer: Principles of Dental Imaging - Page 55, Olaf E. Langland, Robert P. Langlais,
John W. Preece,2nd e
Hunters and Driffield curve :sensitometric curve of radiographic film, a plot of the film density
versus the logarithm of the relative exposure.

146.True congenital macroglossia occur in all except?

a.Beckwidth wiedmann syndrome


b.Renu-osler weber syndrome

c.Behmel syndrome

d.Tollner syndrome

Ans: b.Renu-osler weber syndrome

Refer: Shafer's Textbook of Oral Pathology - Page 29, By Arya Rajendran, B


Sivapathasundharam/7th e

True congenital macroglossia:

Beckwidth wiedmann syndrome

Behmel syndrome

Tollner syndrome

Macroglossia

Prenatally diagnosed
Beckwith-Wiedemann4,10,14,15
Observed in newborns
Beckwith-Wiedeann 1-4,11,12,15,24-30

ancyloglossia superior31

aspartylglucosaminuria23

autosomal dominant inheritance23,32

Behmel syndrome23

trisomy 2134,35,38

lethal dwarfism Blomstrand23

microcephaly-hamartoma
Wiedemann23,36

mucopolysaccharidosis37

muscular idiopathic hypertrophy38-41

neoplasms13,18,20,34,42-45

skeletal dysplasia Urbach23

thyroid tissue persistence35

Tollner syndrome23
Trisomy 2246

variation of normal35
Observed later in life
acromegaly19,41,47

glycogenosis type IIa41

hypothyroidism3,18

inflammatory disease41

Laband syndrome41

multiple hemangiomas Segelman23

myxoedema41

mucolipidosis20

neurofibromatosis20

plasmacytoma41

postoperative complication49

primary amyloidosis18,21,50

147.Highest tendency for cervical enamel projection is found in ?

a.Maxillary 1st molar

b.Maxillary 2nd molar

c.Mandibular premolars

d.Mandibular 1st molar

Ans: d.Mandibular 1st molar

Refer: Mosby's Review for the NBDE - Part 1 - Page 307

The incidence of cervical enamel projections varied between molars. The mandibular second
molars showed the highest incidence of enamel projections (51.0%), followed by the
maxillary second molars (45.6%). The lowest incidence was seen in the maxillary first molars
(13.6%).

148.Maximum chance of sialolithiasis occus in?

a.sublingual gland
b.Parotid duct

c.Submandibular gland

d.Parotid gland

Ans: c.Submandibular gland

Refer: Shafer'S Textbook Of Oral Pathology (6Th Edition) - Page 544

Sialolithiasis accounts for about 50% of all disease occurring in major salivary glands, and
for about 66% of all obstructive salivary gland diseases. About 85% of stones occur in the
submandibular gland, and between 5-10% occur in the parotid gland. In about 0-5% of
cases, the sublingual gland or a minor salivary gland is affected. When minor glands are
rarely involved, caliculi are more likely in the minor glands of the buccal mucosa and the
maxillary labial mucosa.

149.Raindrop appearance is due to?

a.Bismuth

b.Arsenic

c.Mercury

d.Lead

Ans: b.Arsenic

Refer: Shafer'S Textbook Of Oral Pathology (6Th Edition) - Page 553

Chronic arsenic exposure through drinking water leads to an increase in mortality which is
directly related to the dose of arsenic exposure.

Main clinical features and possible complications

Skin lesions:
o Skin lesions are the most common effect of chronic exposure.

o They typically start about ten years after first exposure.

o Keratoses on the palms and soles are characteristic.

o Mees' lines (transverse white lines on nails).

o Hyperpigmentation (especially on the arms and upper chest) - diffuse dark


areas or 'raindrop' pigmentation.

o Also, exfoliative dermatitis, alopecia, conjunctivitis, corneal ulceration.

GI:
o Anorexia, weight loss, abdominal pain, diarrhoea.

o Jaundice, hepatomegaly; development of portal fibrosis and cirrhosis.

Cardiac/respiratory:

o Coronary heart disease, increased further in those who also smoke.

o Myocarditis, pericarditis.

o Hypertension.

o Peripheral arterial disease.

o Restrictive or obstructive lung disease.

o Effects in children are seen after an average of seven years of exposure.

Haematological:

o Pancytopenia,

o Aplastic anaemia.

Neurological:

o Peripheral neuropathy (sensory and motor at 1-3 weeks).

o Muscle fasciculation and wasting.

o Ataxia.

Diabetes - increased risk.

Cancer - increased risk of cancers:

o Basal cell carcinoma and squamous cell carcinoma of the skin.

o Lung.

o Bladder and kidney.

o Possibly also lymphoid, laryngeal and liver.

150.Hyperextension of tongue is seen in?

a.Ehlers danlos syndrome

b.Ectodermal dysplasia

c.Marphans syndrome

d.Beckwith weidmann syndrome

Ans: a.Ehlers danlos syndrome

Refer: Shafer 7th ed 806/6th ed 798


The classic signs of EDS are joint hypermobility; hyperelasticity of skin, which is soft, thin
and fragile; the presence of dystrophic scars; and a tendency to excessive bleeding
manifested by bruises, ecchymoses and hematomas. At least 15 phenotypes of the
syndromes have been catalogued to date, with 2 having recently been reclassified. We
present a description of the first 8 of these phenotypes, but recommend that you read the
works of Pope and Gorlin for further information.

Type I

In type I the skin is hyperelastic. Bony prominences such as the forehead, chin, elbows,
knees and ankles are constantly lacerated. Given the limited healing power of the skin with
this condition, pigmented and atrophic scars (cigarette paper scar) are frequently found in
these areas and they are accompanied at times by fibrous nodules caused by the fibrous
transformation of subcutaneous hematomas.

The patient usually has a mesomorphic appearance with hands and feet being slightly larger
than average. Along with a generalized hypermobility of the joints there is usually an
abnormal bleeding tendency. Occasionally the syndrome is accompanied by mitral valve
prolapse.

Over half of pregnant women with the condition give birth prematurely, following rupture of
the fetal membranes.

Type II

Similar to type I, this form is less severe clinically. The scarring is less common, bleeding
tendency is less pronounced and nodules are much smaller or totally absent. However, joint
hypermobility is similar to that observed in type I and premature births are also a feature of
type II, even if they are less common.

Type III

The patient with type III is usually tall and thin, as in Marfans syndrome. The dominant
features of this variation are joint hypermobility and hyperelasticity of the skin, which often
feels velvety. Bruises and dystrophic scars are rarely observed.

Type IV

Type IV is characterized by a marked fragility of the vascular system. The individual is


usually smaller than average. The skin of the hands and of the back appears to age
prematurely (acrogeria). The eyes are widely spaced, the nose is narrow, the ear lobes are
atrophied and the hair is thin. Aneurysms of the large and medium-sized arteries (axillary,
femoral and carotid) are common. Frequently scar tissue appears to be hemorrhagic.
Rupture of internal organs is common even at a young age, and perforation of the intestinal
tract can be a problem. Joint hypermobility is usually limited to the small articulations of the
hand, with acro-osteolysis of the distal phalanges. Mitral valve prolapse can also be present.

Rupture of arteries and hollow organs such as the uterus and the intestine explains the high
mortality rate (51% before age 40). Type IV is consequently, the most serious form of EDS.

Type V
Women are only carriers of this rare form, similar to types I and II, because transmission is
associated with the X chromosome.

Type VI

This form resembles type I except for an ocular involvement with detached retina being a
common feature. Severe scoliosis and vascular rupture are also features.

Type VII

This is a rare phenotype characterized by joint hypermobility, bilateral dislocation of the hips
and a small stature.

Type VIII

First described by Stewart and others in 1977,type VIII is characterized by generalized early-
onset periodontitis and by large patches of scar tissue on the shins, similar to diabetic ulcers
or varicose veins. The periodontal problems appear at puberty and usually lead to loss of the
teeth before age 30. Biesecker and others describe a case of a man who became
edentulous at age 16 as a result of severe periodontitis. Hoffman and other describe a case
of EDS type VIII where a girl required splinting to treat mobile teeth. Type I (Hoffman) and
type III (Lapire and Nusgens) anomalies of collagen have been linked to EDS type VIII.

Hyperelasticity of the skin and hypermobility of the joints are moderate in this phenotype.
The facies can resemble that described in type IV hypertelorism, widening of the root of
the nose, a narrow curved nose, narrow face and scarring on the forehead and chin.

Clinical Manifestations of EDS

Extraoral

The extraoral manifestations of EDS are the presence of scarring on the chin and forehead,
a history of repeated luxations of the TMJ, epicanthus, hypertelorism, a narrow curved nose,
sparse hair and hyperelasticity of the skin.

Intraoral

The classic intraoral signs of EDS can point to the eventual diagnosis of the condition.

Mucosa

As fragile as the skin, the mucosa tears easily when touched by instruments. Sutures do not
hold.

Periodontal Tissues

The fragility of the gingiva can be detected following treatments such as prophylaxis,
periodontal surgery or extraction. Hemorrhage may be difficult to control during surgical
procedures. Early-onset generalized periodontitis is one of the most significant oral
manifestations of the syndrome. This can lead to the premature loss of deciduous and
permanent teeth.

The Teeth
Hypoplasia of the enamel is commonly seen.Premolar and molar teeth can present with
deep fissures and long cusps. The teeth seem to be fragile and microdontia is sometimes
present. Radiographic examination often reveals pulp stones and roots that are short and
deformed. Microscopic-level anomalies of the various dental tissues are described in detail
by Barabas and Pope. One case of type III EDS with multiple supernumerary teeth has been
reported in the literature.

The Tongue

The tongue is very supple. Approximately 50% of those with the syndrome can touch the end
of their nose with their tongue (Gorlins sign), compared to 8-10% of the population.

The Palate

The palate is commonly vaulted.

151.While taking a radiograph of upper 1st premolar its buccal root appear distal to lingual
root.What should be the angulation of cone for this positioning?

a.Greater vertical angulation

b.Cone head was more mesial

c.Lesser vertical angulation

d.Cone head were placed from more mesial

Ans: b.Cone head was more mesial

Refer:White and pharaoh /6th e Ingle's Endodontics 6 - Page 560

While taking a radiograph of upper 1st premolar its buccal root appear distal to lingual root ,
angulation of cone for this positioning is Cone head was more mesial.

152.Highest pigmentation occur in blacks at?

a.Hard palate

b.gingiva

c.Buccal mucosa

d.Tongue

Ans: a.Hard palate

Refer:Carranza s 11th e ,page no 25

Highest pigmentation occur in blacks at Hard palate


RADIOLOGY

153.Least time required to radiographically diagnose a hard tissue union after tooth fracture?

a.1 months

b.2months

c.3 months

d.4 months

Ans: d.4 months

Refer:White and pharaoh 6th e /page no 544

Least time required to radiographically diagnose a hard tissue union after tooth fracture 4
months .

154.Maxillary sinusitis is radiographically checked by?

a.Fluid level

b.Clouding and air fluid level

c.clouding of antrum

d.Clouding and bony erosion

Ans: b.Clouding and air fluid level

Refer: Essentials of Oral Pathology - Page 423, Swapan Kumar Purkait,6th e

Maxillary sinusitis is radiographically checked by Clouding and air fluid level. Peripheral
mucosal thickening, air/fluid level, air bubbles within the fluid and obstruction of the
ostiomeatal complexes are recognised findings.

155.In an IOPA ,mesial to mandibular 1st molar about 2mm of bone loss was invisible
.However on probing pocket depth is about 6mm .This difference is due to?

a.Poor angulation of x-ray


b.Thickness of buccal and lingual cortical plates in that region mask the bone loss.

c.Presence of tori in that region mask the loss

d.Loss of cortical plates over the defects.

Ans: b.Thickness of buccal and lingual cortical plates in that region mask the bone loss.

Refer:White & pharaoh ,page no 168,6th e

Vertical defects are difficult to recognize on a radiograph because one or both of the cortical
plates remain superimposed over the defect.Clinical and surgical means are the best means
of detecting.

156.In a radiograph ,a thin radiolucent line seen between central and lateral incisor is most
probably?

a.An artefact

b.A nutrient canal

c.A external fistula

d.Lamina dura

Ans: b.A nutrient canal

Refer: Essentials of Dental Radiology - Page 88, Pramod John R.,3rd e


Nutrient canals are anatomic structures of the alveolar bone through which neurovascular
elements transit to supply teeth and supporting structures. A dental identification using a
nutrient canal of the mandibular alveolar process as the most compelling anatomic feature
for antemortem-postmortem radiographic comparison is described. Nutrient canals as a
potential marker for clinical disease.

ORTHODONTICS
157. Which fontanelle closes the last :
A.anterior
B.posterior
C.postrolateral
D.Anteriolateral
Ans: A.anterior
Refer: Paediatric Dentistry - Page 9, Richard Welbury, Monty S. Duggal, Marie
Thrse Hosey,1st e

The ossification of the bones of the skull causes the anterior fontanelle to close over by 9 to
18 months.The sphenoidal and posterior fontanelles close during the first few months of life.
The closures eventually form the sutures of the neurocranium. Other than the anterior and
posterior fontanelles, the mastoid fontanelle and the sphenoidal fontanelle are also
significant.

158.If the facial index of patient is 85.2 than his face will be?

a.Euproscopic

b.Mesoproscopic

c.Hyperprosic

d.Leptoproscopic

Ans: b.Mesoproscopic

Refer: Textbook of Orthodontics - Page 64, Gurkeerat Singh,3rd e

Mesoprosopic : having a face of average width with a facial index of 84.0 to 87.9 as
measured on the living head or 85.0 to 89.9 on the skull

159.There are two pictures shown below.According to Ackerman classification of normal


spontaneous smile or posed smile which is true?

Sp1 Sp2

a.sp1 is posed smile

b.sp2 is posed smile


c.Both are posed smile

d.Both are spontaneous smile

Ans: a.sp1 is posed smile

Refer: Kharbanda ,page no.160, 1st e

Ackerman et al.1 among others have defined two main types of smiles: social smiles and
enjoyment smiles. A social smile is the voluntary smile a person uses in social settings or
when posing for a photograph. The social smile is posed, which means that it is not
elicited or accompanied by emotion. This type of smile can be sustained as a static facial
expression and does not appear strained.2 On the other hand, enjoyment smiles are
involuntary and are elicited by laughter. The enjoyment smile is unposed and reflects the
emotion that one is experiencing at that moment. This smile appears strained because the
mouth bursts

160.Frankfort Horizontal plane is commonly used in different cephalometric measurement


.This name is given on basis of?

a.According to birthplace of anthropologist who discovered it Frankfort ,Germany.

b.On the name of scientist himself W.H .Frankfort

c.On the name of university of Frankfort where several research on this.

d.On the name of place where conference of anthropologist held to discuss it in details.

Ans: d.On the name of place where conference of anthropologist held to discuss it in details

Refer: Textbook of Orthodontics - Page 183, Gurkeerat Singh/3rd e

Frankfort Horizontal plane is commonly used in different cephalometric measurement .This


name is given on basis of On the name of place where conference of anthropologist held to
discuss it in details.
161.In many facial measurement procedures,ratio of is middle third face height to lower third
face height taken.In the consideration ,what should be the ratio of middle third (g-Sn)to lower
third(Sn-Mn)?

a.1:1.5

b.1:1

c.1:2

d.2:1

Ans: b.1:1

Refer:Bhalaji 4th e ,page no. 139

1:1 should be the ratio of middle third (g-Sn)to lower third(Sn-Mn).

162.In given orthodontic model,a wire is shown adapted over the teeth .It shows which order
of bends?

a.First order only

b.First and second order

c.Second order only

d.All first ,second and third order bends


Ans: a.First order only

Refer:Gurkeerat singh 2nd e /page no 468-469

First Order - bends that are in the occlusal plane. Step in, step out, rotation bends Hold
the pliers perpendicular to the occlusal plane

Other Types of First Order Bends Bayonet bends Toe in bends Other Types of Second
Order Bends Tie-back bends Reverse curves Closing loops Headgear adjustment
loops Retainer adjustment loops

Second Order - bends that are in the vertical dimension. Step up, step down, tip bends
Hold the pliers level with to the occlusal plane

Third Order - bends that are in the occlusal plane.

163.How much overjet can be corrected by orthopaedic appliance?

a.0-7mm

b.5-7mm

c.12-13mm

d.12-17mm

Ans: c.12-13mm

Refer:Profit 4th e ,page no.690

12-13mm overjet can be corrected by orthopaedic appliance.

164.According to Ricketts analysis classification ratio of maxillary to mandibular arch width


is?

a.60%

b.70%

c.80%
d.90%

Ans: c.80%

Refer:Kharbanda ,1st e ,page no .135

According to Ricketts analysis classification ratio of maxillary to mandibular arch width is


80%.

165.Following instrument shown in the figure is :

a.Dontrix Gauge

b.MBT/Will gauge

c.Dynometer

d.Boon gauge

Ans: d.Boon gauge

Refer: Orthodontics: Principles and Practice - Page 340, Basavaraj Subhashchandra


Phulari/1st e

Helps with precise bracket placement. Accurately measures height of bracket from edge.
3.5, 4, 4.5, 5mm. Made from surgical stainless steel. Autoclavable.

166.With increasing bone loss center of resistance of a tooth shifted?

a.Towards incisal

b.More apically
c.At infinite

d.Out of teeth

Ans: b.More apically

Refer: Biomechanics in Clinical Orthodontics - Page 298, Ravindra Nanda,1st e

In cases of adult patients with horizontal bone loss, since the applied force in the crown is
transferred to the apices with greater magnitude, since the center of resistance of teeth, in
such situations, are more apically located.

167.Which of the following is not used for anterior and posterior relationship of maxillary and
mandibular anterior teeth?

a.ANB

b.Facial axis

c.Facial angle

d.Na-Pog perpendicular

Ans: b.Facial axis

Refer: Orthodontics: Diagnosis and Management of Malocclusion and Dentofacial


complex/By Kharbanda/page no 379,1st e

Facial axis not used for anterior and posterior relationship of maxillary and mandibular
anterior teeth.

168.In the following case of elastic ribbon is used to retract upper anterior teeth with cortical
device as anchorage .Which type of anchorage is it?

a.Stationary anchorage

b.Absolute anchorage

c.Reinforced anchorage

d.Cortical anchorage
Ans: b.Absolute anchorage

Refer: Textbook of Orthodontics - Page 273, Gurkeerat Singh,3rd e

In Absolute anchorage elastic ribbon is used to retract upper anterior teeth with cortical
device as anchorage.

169.Following clasp design is also known as?

a.South end clasp

b.Shwartz clasp

c.Crozat clasp

d.Delta clasp

Ans: c.Crozat clasp

Refer: Textbook of Orthodontics - Page 447,3rd e , Gurkeerat Singh

Crozat clasps are typically used on first permanent molars as part of a Crozat appliance.
They consist of three parts. The main body of the clasp, called the crib, is formed from one
piece of wire and is tightly adapted around the entire tooth. The second wire, called the
crescent, provides the retention by engaging the undercuts of the proximal surfaces. The
third part of the clasp is the occlusal rest. It passes through the lingual groove and rests
tightly on the occlusal surface.

170.Following appliance is used for?

a.Treat posterior crossbite

b.Rapid maxillary expansion

c.Treat anterior as well as posterior cross bite

d.Correction of midline diastemma

Ans: a.Treat posterior crossbite

Refer: TEXTBOOK OF ORTHODONTICS - Page 457, Sridhar Premkumar,1st e

The quad helix is more flexible version of the W-arch .The helices in the anterior plate are
bulky ,which can effectively serve as a reminder to aid in stopping a finger habit.This
combination of a posterior crossbite and a finger sucking habit is the best indication for this
appliance.

171.A 9 year old patient came to your dental clinic with facial asymmetry and with constricted
maxilla what will be your first step recommended?

a.Take the x-ray

b.Refer to orthodontist

c.Reassure to parents and wait till all permanent teeth erupts.

d.Perform maxillary expansion with removale appliance

Ans: b.Refer to orthodontist

Refer:Profitt 4 th e ,page no 164

9 year old patient came to your dental clinic with facial asymmetry and with constricted
maxilla first step recommended is refer to orthodontist.

172.Following instrument is used for advancement of maxilla also known as?

a.Delire face mask


b.Petit face mask

c.High pull head

d.McNamara Appliance

Ans: b.Petit face mask

Refer: Textbook of Orthodontics - Page 513,3rd e , Gurkeerat Singh


Petit face mask is used for the advancement of maxilla.

173.In the following diagram Scammons growth curves of various body tissues are
shown.Arrow marked line(dotted line at 100% postnatal growth here) shows growth of?

a.Lymphoid tissue

b.Brain tissue

c.Mandibular tissue

d.General tissue
Ans: b.Brain tissue

Refer: Orthodontics: Principles and Practice - Page 28, Basavaraj Subhashchandra


Phulari.1st e

Arrow in exam was given on brain and neural tissue.

174.Katz classification is based on?

a.Incisor

b.Canine

c.Premolar

d.Molar

Ans: c.Premolar

Refer: Orthodontics: Diagnosis and Management of Malocclusion and Dentofacial


complex,By Kharbanda,page no 43,1st e
Katz classification is based on premolars.

PCD &PCM
175.All are true about RCT except?

a.Interviewer bias can be reduced by double blind study.

b.Dropout should be excluded from study

c.Sample size depends on type of study.

d.Reduce selection bias in allocation of treatment.

Ans: A Textbook of Public Health Dentistry - Page 18, CM Marya,1st e

randomized controlled trial (or randomized control trial;RCT) is a type of scientific (often
medical) experiment, where the people being studied are randomly allocated one or other of
the different treatments under study. The RCT is often considered the gold standard for a
clinical trial. RCTs are often used to test the efficacy or effectiveness of various types of
medical intervention and may provide information about adverse effects, such as drug
reactions. Random assignment of intervention is done after subjects have been assessed for
eligibility and recruited, but before the intervention to be studied begins. Random allocation
in real trials is complex, but conceptually, the process is like tossing a coin. After
randomization, the two (or more) groups of subjects are followed in exactly the same way,
and the only differences between the care they receive, for example, in terms of procedures,
tests, outpatient visits, and follow-up calls, should be those intrinsic to the treatments being
compared. The most important advantage of proper randomization is that it minimizes
allocation bias, balancing both known and unknown prognostic factors, in the assignment of
treatments. Dropout should be excluded from study.Sample size depends on type of study.
Reduce selection bias in allocation of treatment.

ideal randomization procedure would achieve the following goals:

Maximize statistical power, especially in subgroup analyses. Generally, equal


group sizes maximize statistical power, however, unequal groups sizes maybe more
powerful for some analyses (e.g., multiple comparisons of placebo versus several
doses using Dunnetts procedure ), and are sometimes desired for non-analytic
reasons (e.g., patients maybe more motivated to enroll if there is a higher chance of
getting the test treatment, or regulatory agencies may require a minimum number of
patients exposed to treatment).
Minimize selection bias. This may occur if investigators can consciously or
unconsciously preferentially enroll patients between treatment arms. A good
randomization procedure will be unpredictable so that investigators cannot guess the
next subject's group assignment based on prior treatment assignments. The risk of
selection bias is highest when previous treatment assignments are known (as in
unblinded studies) or can be guessed (perhaps if a drug has distinctive side effects).

Minimize allocation bias (or confounding). This may occur when covariates that
affect the outcome are not equally distributed between treatment groups, and the
treatment effect is confounded with the effect of the covariates (i.e., an "accidental
bias"). If the randomization procedure causes an imbalance in covariates related to
the outcome across groups, estimates of effect may be biased if not adjusted for the
covariates (which may be unmeasured and therefore impossible to adjust for).
176.The recommended water fluoride level in a middle school water fluoridation plan should
be?

a.1ppm

b.2ppm

c.4ppm

d.12ppm

Ans: c.4ppm

Refer: A Textbook of Public Health Dentistry - Page 346, CM Marya,1st e

The recommended water fluoride level in a middle school water fluoridation plan should be
4ppm.

177.Range of Bristle range of adult tooth brush is?

a.0.007-0.015inch

b. 0.007-0.015inch

c. 0.007-0.015inch

d. 0.007-0.015inch

Ans: a.0.007-0.015inch

Refer: Textbook of Preventive and Community Dentistry - Page 264, S. S. Hiremath,2nd


e

The diameter for adults ranges from 0.007-0.015 inches.


For children: 0.005 and short bristles.
Soft: 0.007-0.009.
Medium: 0.010-0.012.
Hard: 0.013-0.014.

178.In a community trial if we select every 5 th house in a village ,the sampling is called as?

a.Stratified random sampling

b.Cluster sampling

c.Stratified sampling
d.Systemic random sampling

Ans: d.Systemic random sampling

Refer: Donaldsons' Essential Public Health - Page 66,3rd e

With the systematic random sample, there is an equal chance (probability) of selecting
each unit from within the population when creating the sample. The systematic sample is
a variation on the simple random sample. Rather than referring to random number tables to
select the cases that will be included in your sample, you select units directly from the
sample frame. With systematic random sampling, there would an equal chance
(probability) that each of the 10,000 students could be selected for inclusion in our sample.
Each of the 10,000 students is known as a unit, a case or an object (these terms are
sometimes used interchangeably; we use the word unit).

179.Which type of waste is disposed in the yellow container?

a.Contaminated and to be incerated.

b. Contaminated and non incerated.

c.Sharp

d. Uncontaminated and to be incerated.

Ans: a.Contaminated and to be incerated.

Refer: Microbiology for Dental Students - Page 310, B.S. Nagoba,1st e

Option Treatment & Disposal Waste Category

Cat. No. 1 Incineration /deep Human Anatomical Waste (human tissues, organs,
burial body parts)

Cat. No. 2 Incineration /deep Animal Waste Animal tissues, organs, Body parts
burial carcasses, bleeding parts, fluid, blood and
experimental animals used in research, waste
generated by veterinary hospitals /
colleges,discharge from hospitals, animal houses)

Cat. No. 3 Local autoclaving/ micro Microbiology & Biotechnology waste (wastes from
waving/ incineration laboratory cultures, stocks or specimens of micro-
organisms live or attenuated vaccines, human and
animal cell culture used in research and infectious
agents from research and industrial
laboratories,wastes from production of biological,
toxins, dishes and devices used fortransfer of
cultures)

Cat. No. 4 Disinfections (chemical Waste Sharps (needles, syringes, scalpels blades,
treatment glass etc. that may cause puncture and cuts. This
/autoclaving/micro includes both used & unused sharps)
waving and mutilation
shredding

Cat. No. 5 Incineration / Discarded Medicines and Cytotoxic drugs (wastes


destruction & drugs comprising of outdated, contaminated and discarded
disposal in secured medicines)
landfills

Cat. No. 6 Incineration , Solid Waste (Items contaminated with blood and
autoclaving/micro body fluids including cotton, dressings, soiled plaster
waving casts, line beddings, othermaterialcontaminated
with blood)

Cat. No. 7 Disinfections by Solid Waste (waste generated from disposable items
chemical treatment other than the waste sharps such as tubing,
autoclaving/micro catheters, intravenous sets etc.)
waving& mutilation
shredding.

Cat. No. 8 Disinfections by Liquid Waste (waste generated from laboratory &
chemical treatment washing, cleaning , house-keeping and disinfecting
and discharge into drain activities)

Cat. No. 9 Disposal in municipal Incineration Ash (ash from incineration of any bio-
landfill medical waste)
Cat. No. 10 Chemical treatment &
discharge into drain for Chemical Waste (chemicals used in production of
liquid & secured landfill biological, chemicals, used in disinfect ion, as
for solids insecticides, etc)

Option Treatment & Disposal Waste Category

Chemicals treatment using at least 1% hypochlorite solution or any other equivalent


chemical reagent. It must be ensured that chemical treatment ensures disinfections.
Mutilation/shredding must be such so as to prevent unauthorised reuse.
There will be no chemical pretreatment before incineration. Chlorinated plastics shall not
be incinerated.
Deep burial shall be an option available only in towns with population less than five lakhs
and in rural areas.
The most essential part of hospital waste management is the segregation of Bio-medical
waste. The segregation of the waste should be performed within the premises of the
hospital/nursing homes. The color coding, type of container to be used for different waste
category and suggested treatment options are listed below.

COLOR CODING & TYPE OF CONTAINER FOR DISPOSAL OF BIO-MEDICAL WASTE


Colour Type of Waste Category Treatment Options as per
Coding Containers Schedule 1

Yellow Plastic bag 1,2,3,6 Incineration/deep burial

Red Disinfected 3,6,7 Autoclaving/Micro waving/


Container/ Plastic Chemical Treatment
bag

Blue/ White Plastic 4,7 Autoclaving/Micro waving/


translucent bag/puncture chemical treatment and
proof container destruction/shredding

Black Plastic bag 5,9,10 (Solid) Disposal in secured landfill

Notes:
1. Color coding of waste categories with multiple treatment options as defined in
Schedule 1, shall be selected depending on treatment option chosen, which shall be
as specified in Schedule 1.
2. Waste collection bags for waste types needing incineration shall not be made
of chlorinated plastics.
3. Categories 8 and 10 (liquid) do not require containers/bags.
4. Category 3 if disinfected locally need not be put in Containers/bags.
Different labels for Bio-medical waste containers and bags shall be required for identification
and safe handling of this waste. These labels for storage/transportation of Biomedical waste
are as under,:

LABEL FOR BIO-MEDICAL WASTE CONTAINERS/BAGS

180.Study to check incidence of caries in 12 year old students in the state of Madhya
Pradesh ?

a.Prospective

b.Retrospective
c.observational study

d.Descriptive study

Ans: a.Prospective

Refer: Epidemiology for Public Health Practice - Page 403, Robert H. Friis, Thomas
Sellers,5th e

Prospective

A prospective study watches for outcomes, such as the development of a disease, during the
study period and relates this to other factors such as suspected risk or protection factor(s).
The study usually involves taking a cohort of subjects and watching them over a long period.
The outcome of interest should be common; otherwise, the number of outcomes observed
will be too small to be statistically meaningful (indistinguishable from those that may have
arisen by chance). All efforts should be made to avoid sources of bias such as the loss of
individuals to follow up during the study. Prospective studies usually have fewer potential
sources of bias and confounding than retrospective studies.

Retrospective

A retrospective study looks backwards and examines exposures to suspected risk or


protection factors in relation to an outcome that is established at the start of the study. Many
valuable case-control studies, such as Lane and Claypon's 1926 investigation of risk factors
for breast cancer, were retrospective investigations. Most sources of error due to
confounding and bias are more common in retrospective studies than in prospective studies.
For this reason, retrospective investigations are often criticised. If the outcome of interest is
uncommon, however, the size of prospective investigation required to estimate relative risk is
often too large to be feasible. In retrospective studies the odds ratio provides an estimate of
relative risk. You should take special care to avoid sources of bias and confounding in
retrospective studies.

Prospective investigation is required to make precise estimates of either the incidence of an


outcome or the relative risk of an outcome based on exposure.

Case-Control studies

Case-Control studies are usually but not exclusively retrospective, the opposite is true for
cohort studies. The following notes relate case-control to cohort studies:

outcome is measured before exposure

controls are selected on the basis of not having the outcome


good for rare outcomes

relatively inexpensive

smaller numbers required

quicker to complete

prone to selection bias

prone to recall/retrospective bias

related methods are risk (retrospective), chi-square 2 by 2 test, Fisher's exact test,
exact confidence interval for odds ratio, odds ratio meta-analysis and conditional
logistic regression.

Cohort studies

Cohort studies are usually but not exclusively prospective, the opposite is true for case-
control studies. The following notes relate cohort to case-control studies:

outcome is measured after exposure

yields true incidence rates and relative risks

may uncover unanticipated associations with outcome

best for common outcomes

expensive

requires large numbers

takes a long time to complete

prone to attrition bias (compensate by using person-time methods)

prone to the bias of change in methods over time

related methods are risk (prospective), relative risk meta-analysis, risk difference
meta-analysis and proportions

181.Some investigators found association between obesity and dental caries. For this
study ,data was directly taken from some hospital records and food samples from some
specific private company. Now they wanted to train children for the same ,this study an
example of?

a.Experimental study
b.Ecological study

c.Cross sectional /crossover

d.RCT

Ans: c.Cross sectional /crossover

Refer:Essential Dental Public Health - Page 248, Blnaid Daly, Paul Batchelor,
Elizabeth Treasure,2nd e

Cross-Sectional Survey is an observational study where the researchers observe what is


happening without interfering. Data is collected at a specific point in time and often used to
assess the prevalence of an illness or disease. Ex. Association between awareness of skin
health during rehab and prevalence of pressure sores five years post injury.

Case-Control Study is an observational study where two groups are identified and divided
based on whether or not they have a specific disease or condition (cases vrs. control). The
researcher then explores retrospectively (i.e. back in time) for clues (i.e. exposure to certain
risk factors) as to the cause of that disease or condition. Ex. Case-Control study determined
that people with SCI who suffered from depression lacked a strong network of support.

Cohort Study is an observational study in which a group of individuals a cohort are


followed over a period of time in order to determine either the incidence rate of certain
diseases or the risk factors that lead to those diseases. These studies can be either
prospective or retrospective. In prospective studies, the cohort of individuals are divided into
two groups based on exposure to a specific risk factor and then followed over a period of
time to determine how many people from each group develop the disease (incidence rate).
In retrospective studies, the cohort is selected from the past and data is collected either in
the past or present on the number of individuals who were exposed to a certain risk factor
and who developed the disease. Ex. People with SCI are followed 10 years post-injury to
determine incidence of secondary complications.

Ecologic Study is a study that looks at the rates of exposure to risk factors and the incidence
of an illness or disease in different geographical areas or time frames. Ex. Incidence of
people with SCI receiving homecare by XXX agency in XXX city that contracted MRSA (a
bacterial infection).

Randomized Controlled Trial (RCT) is a study that is used to test the efficacy or
effectiveness of a specific, treatment, therapy, or medication. RCTs are considered to be the
Gold Standard in clinical trials. The design involves the creation of two distinct groups the
treatment and the no treatment groups. Participants in the study are randomly assigned
to one group or the other. Individuals in the treatment group receive a particular treatment
for example, a pill while the individuals in the no treatment (control) group receive nothing,
or in drug studies receive a placebo a fake pill which has no medicinal qualities. At the
end of the study, the outcomes of the two groups can be measured and the treatment
evaluated. Most studies are double-blinded, meaning that neither the individuals participating
in the study, nor the researchers know to which group the participants belong during the
evaluation period. RCT drug studies typically go through 4 levels of phases before getting
approved by the FDA:
Phase I trials usually involve a small number of participants on whom the therapy is tested in
a range of doses or treatments. The objective is to establish a dose range that is well-
tolerated, safe, and effective for therapeutic use.

Phase II trials usually involve a small number of participants and they establish whether the
therapy is effective at safe and optimal doses of the drug.

Phase III trials usually involve a large number of participants and compare the new therapy
to other therapies and/or to a placebo. These studies are used to provide specific evidence
of efficacy and common side effects, as well as a determiner for drug licensing.

Phase IV trials trials usually involve a very large number of participants and are performed
after a drug is approved and available to the general public. These studies are conducted to
detect the rate or existence of uncommon side effects.

Cross-Over Design studies are another form of RCT. These studies involve participants
being randomized into two groups, a treatment or no treatment group, and outcomes are
compared, followed by a washout period and then participants are switched to the other
group with outcomes being compared. Often, the participants and researcher are blinded as
to which group the participants are in. These studies can only be conducted in cases where
the treatment or therapy does not have a permanent effect on the participant.

182.Adoloscent age is?

a.8-12

b.14-28

c.10-14

d.8-14

Ans: b.14-28

Refer: Pediatric Dentistry: A Clinical Approach - Page 348, Goran Koch, Sven
Poulsen/1st e

Adolescent age is 14-28.

183.Dentist Act was introduced in india in ?

a.1942

b.1946

c.1950

d.1956

Ans: c.1950
Refer: Textbook of Preventive and Community Dentistry - Page 288/3rd e , S. S.
Hiremath

Dentist Act was introduced in india in 29, March 1948.

184.A statistical measure of the degree to which two variables move together?

a.Certainity Equivalent

b.Variance

c.Covariance

d.Coefficient of variance

Ans: c.Covariance

Refer: Essential Statistical Methods for Medical Statistics - Page 233, J. Philip
Miller/1st e

A statistical measure of the degree to which two variables move together Covariance

185.Simplest method commonly used measure of assessment of variation in a sample is?

a.SD

b.Mean

c.Median

d.Range

Ans: d.Range

Refer: Principles of Public Health Practice - Page 402, F. Douglas Scutchfield, C.


William Keck,2nd e

Simplest method commonly used measure of assessment of variation in a sample is


Range. The range is simple to compute and is useful when you wish to evaluate the
whole of a dataset.
The range is useful for showing the spread within a dataset and for comparing the
spread between similar datasets.

186.If a confidence interval is 99.67 then it includes?


a.1SD

b.2SD

c.3SD

d.4SD

Ans:c

Refer: Community Oral Health Practice for the Dental Hygienist, Kathy Voigt
Geurink,page no.215,3rd e

In statistics, the so-called 689599.7 rule is a shorthand used to remember the percentage
of values that lie within a band around the mean in a normal distribution with a width of one,
two and three standard deviations, respectively; more accurately, 68.27%, 95.45% and
99.73% of the values lie within one, two and three standard deviations of the mean,
respectively. In mathematical notation, these facts can be expressed as follows, where x is
an observation from a normally distributed random variable, is the mean of the distribution,
and is its standard deviation:

In the empirical sciences the so-called three-sigma rule of thumb expresses a


conventional heuristic that "nearly all" values are taken to lie within three standard deviations
of the mean, i.e. that it is empirically useful to treat 99.7% probability as "near certainty". The
usefulness of this heuristic of course depends significantly on the question under
consideration, and there are other conventions, e.g. in the social sciences a result may be
considered "significant

" if its confidence level is of the order of a two-sigma effect (95%), while in particle physics,
there is a convention of a five-sigma effect (99.99994% confidence) being required to qualify
as a "discovery".
187.If a distribution has excessive negative or positive skewness what measure of central
tendency is best for it?

a.Mode

b.Mean

c.Median

d.SD

Ans: c.Median

Refer: Principles of Dental Public Health - Page 107, James Morse Dunning,4th e

If a distribution has excessive negative or positive skewness Median is best used for central
tendency.

The three most commonly-used measures of central tendency are the following.

mean

The sum of the values divided by the number of values--often called the "average."

Add all of the values together.


Divide by the number of values to obtain the mean.

Example: The mean of 7, 12, 24, 20, 19 is (7 + 12 + 24 + 20 + 19) / 5 = 16.4.


median
The value which divides the values into two equal halves, with half of the values
being lower than the median and half higher than the median.

Sort the values into ascending order.


If you have an odd number of values, the median is the middle value.

If you have an even number of values, the median is the arithmetic mean (see
above) of the two middle values.

Example: The median of the same five numbers (7, 12, 24, 20, 19) is 19.
mode

The most frequently-occurring value (or values).

Calculate the frequencies for all of the values in the data.


The mode is the value (or values) with the highest frequency.

Example: For individuals having the following ages -- 18, 18, 19, 20, 20, 20, 21, and 23, the
mode is 20.

You might also like